SlideShare une entreprise Scribd logo
1  sur  69
Télécharger pour lire hors ligne
Talleres de Formaci´n Matem´tica
                    o       a
  Maracaibo, 26 al 31 de julio de 2004




    Resoluci´n de
            o
Problemas Matem´ticos
                a


         Jos´ Heber Nieto Said
            e
Prefacio

    Estas notas constituyen el material de apoyo de un taller para estudiantes
Licenciatura en Matem´ticas dirigido a desarrollar la habilidad para resolver
                         a
problemas.
    Aunque por lo general los problemas juegan un rol importante en cual-
quier curso de matem´tica y la habilidad para resolverlos es un aspecto
                        a
importante de la evaluaci´n, los profesores suelen centrar sus esfuerzos en
                            o
los aspectos t´cnicos espec´
               e             ıficas de su asignatura y no en los aspectos gene-
rales de la resoluci´n de problemas. El objetivo de esta obra en cambio es
                    o
ayudar al lector a desarrollar su habilidad general para resolver problemas.
    Es bueno dejar en claro que el desarrollo de esta habilidad es b´sica-
                                                                         a
mente el resultado del trabajo personal, de la pr´ctica adquirida resolviendo
                                                  a
problemas y de la reflexi´n sobre esa pr´ctica. No es posible convertirse en
                           o               a
un solucionista experto mediante la mera lectura pasiva de un libro, del
mismo modo que no es posible convertirse en un buen nadador o pianista
simplemente leyendo un manual. Sin embargo el conocimiento de las t´cni-  e
cas apropiadas y de los errores t´  ıpicos que es preciso evitar puede ser tan
util para el solucionista como lo es para el nadador o el pianista.
´
    Con el fin de que la obra sea de utilidad para el mayor n´mero posible
                                                                 u
de estudiantes se ha procurado que los problemas analizados no requieran
de conocimientos especializados. Sin embargo las mismas t´cnicas y estra-
                                                               e
tegias que ejemplificamos con problemas elementales se aplican a los m´s     a
avanzados.
´
Indice general

Introducci´n
          o                                                                                                      1

1. Principios Generales                                                                                          3
   1.1. Resoluci´n de Problemas y Creatividad . . .
                o                                                           .   .   .   .   .   .   .   .   .    3
        1.1.1. Invertir el problema . . . . . . . . . .                     .   .   .   .   .   .   .   .   .    4
        1.1.2. Pensamiento lateral . . . . . . . . . .                      .   .   .   .   .   .   .   .   .    4
        1.1.3. Principio de discontinuidad . . . . . .                      .   .   .   .   .   .   .   .   .    5
        1.1.4. Imitaci´n . . . . . . . . . . . . . . . .
                       o                                                    .   .   .   .   .   .   .   .   .    5
        1.1.5. Tormenta de cerebros (Brainstorming)                         .   .   .   .   .   .   .   .   .    5
        1.1.6. Mapas mentales . . . . . . . . . . . .                       .   .   .   .   .   .   .   .   .    5
        1.1.7. Programaci´n neuroling¨´
                            o           uıstica (PNL)                       .   .   .   .   .   .   .   .   .    6
        1.1.8. Factores afectivos . . . . . . . . . . . .                   .   .   .   .   .   .   .   .   .    6
        1.1.9. Bloqueos mentales . . . . . . . . . . .                      .   .   .   .   .   .   .   .   .    6
   1.2. La Creaci´n Matem´tica . . . . . . . . . . . .
                  o          a                                              .   .   .   .   .   .   .   .   .    7
   1.3. La metodolog´ de P´lya . . . . . . . . . . . .
                      ıa      o                                             .   .   .   .   .   .   .   .   .    8
   1.4. El trabajo de Alan Schoenfeld . . . . . . . . .                     .   .   .   .   .   .   .   .   .   11

2. Ejemplos sencillos                                                      14
                     ´
   2.1. Aritm´tica y Algebra . . . . . . . . . . . . . . . . . . . . . . . 14
             e
   2.2. Combinatoria . . . . . . . . . . . . . . . . . . . . . . . . . . . 19
   2.3. Geometr´ . . . . . . . . . . . . . . . . . . . . . . . . . . . . . 21
                ıa

3. Algunas Estrategias B´sicas
                           a                                                                                    26
   3.1. Figuras y diagramas . . . . . .     .   .   .   .   .   .   .   .   .   .   .   .   .   .   .   .   .   26
   3.2. Examen de casos especiales . .      .   .   .   .   .   .   .   .   .   .   .   .   .   .   .   .   .   28
   3.3. Transformaciones e Invariantes      .   .   .   .   .   .   .   .   .   .   .   .   .   .   .   .   .   30
   3.4. El Principio Extremal . . . . .     .   .   .   .   .   .   .   .   .   .   .   .   .   .   .   .   .   34


                                      iii
4. Un     problema y varias soluciones                                                                         37
   4.1.    Inducci´n . . . . . . . . . . . . . .
                  o                                .   .   .   .   .   .   .   .   .   .   .   .   .   .   .   38
   4.2.    Teor´ de grafos . . . . . . . . . . .
               ıa                                  .   .   .   .   .   .   .   .   .   .   .   .   .   .   .   38
   4.3.    Pruebas por Integraci´n . . . . . .
                                 o                 .   .   .   .   .   .   .   .   .   .   .   .   .   .   .   40
   4.4.    El m´todo de perturbaciones . . .
                e                                  .   .   .   .   .   .   .   .   .   .   .   .   .   .   .   41
   4.5.    Funciones escalonadas . . . . . . .     .   .   .   .   .   .   .   .   .   .   .   .   .   .   .   42
   4.6.    Triangulaciones y Lema de Sperner       .   .   .   .   .   .   .   .   .   .   .   .   .   .   .   43

5. Problemas para pensar                                                                                       45

6. Soluciones y sugerencias                                                                                    51

Bibliograf´
          ıa                                                                                                   60




                                        iv
Introducci´n
          o

    La palabra problema proviene del griego πρoβαλλειν, “lanzar adelante”.
Un problema es un obst´culo arrojado ante la inteligencia para ser superado,
                         a
una dificultad que exige ser resuelta, una cuesti´n que reclama ser aclarada.
                                                 o
Todos vivimos resolviendo problemas: desde el m´s b´sico de asegurar la
                                                     a a
cotidiana subsistencia, com´n a todos los seres vivos, hasta los m´s com-
                             u                                        a
plejos desaf´ planteados por la ciencia y la tecnolog´ La importancia de
            ıos                                         ıa.
la actividad de resoluci´n de problemas es evidente; en definitiva, todo el
                         o
progreso cient´ıfico y tecnol´gico, el bienestar y hasta la supervivencia de la
                            o
especie humana dependen de esta habilidad. No es de extra˜ar por lo tanto
                                                              n
que la misma se haya convertido en un nuevo objeto de estudio, atrayendo
por igual la atenci´n de psic´logos, ingenieros, matem´ticos, especialistas
                    o          o                           a
en inteligencia artificial y cient´
                                 ıficos de todas las disciplinas. En el campo
educativo se ha reconocido ampliamente su importancia. y en muchas Uni-
versidades el desarrollo de la creatividad y de la habilidad para resolver
problemas es una parte integral del curriculum.
    Pero lamentablemente todav´ es muy com´n que se expongan ante el
                                  ıa             u
alumno los productos y resultados de la resoluci´n de problemas, pero no el
                                                 o
proceso mismo. Si examinamos un libro de texto con problemas resueltos
de matem´tica, encontraremos por lo general soluciones tersas y acabadas.
          a
Rara vez el autor incluye comentarios sobre los intentos fallidos de soluci´n,
                                                                           o
los casos particulares examinados antes de llegar a la soluci´n general o los
                                                              o
refinamientos realizados a una primera soluci´n no totalmente satisfactoria.
                                               o
Estos y otros elementos del proceso son cuidadosamente eliminados y lo que
se nos presenta es el producto final, conciso y elegante. Hay muchas posibles
razones para que esto sea as´ un estilo de exposici´n matem´tica consa-
                              ı:                       o         a
grado por la tradici´n, criterios est´ticos de concisi´n y elegancia, razones
                     o               e                o
econ´micas de las editoriales, etc. Pero la consecuencia es que el estudiante
     o
obtiene una visi´n falseada de lo que es resolver problemas y de la actividad
                o


                                      1
2


matem´tica en general.
         a
     Si tiene la suerte de tener un profesor que entienda y valore el proceso de
resolver problemas entonces las actividades de aula suplir´n las deficiencias
                                                             a
del texto. Pero si no es as´ y el profesor sigue al libro al pie de la letra, al
                              ı
enfrentarse al primer fracaso el estudiante terminar´ frustrado, perder´ la
                                                        a                   a
confianza en s´ mismo y creer´ que la resoluci´n de problemas es una acti-
                 ı               a                o
vidad incomprensible, accesible solamente a unos pocos superdotados.
     Nuestro principal objetivo en esta obra es ayudar al lector a desarrollar
su habilidad para resolver problemas. Es bueno dejar claro desde el principio
que el desarrollo de esta habilidad es el resultado del trabajo personal, de la
pr´ctica adquirida resolviendo problemas y de la reflexi´n sobre esa pr´ctica.
   a                                                      o               a
No es posible convertirse en un solucionista experto mediante la mera lectura
pasiva de un libro, del mismo modo que no es posible convertirse en un buen
nadador o pianista simplemente leyendo. Sin embargo el conocimiento de las
t´cnicas apropiadas y de los errores t´
 e                                       ıpicos que es preciso evitar puede ser
tan util para el solucionista como lo es para el nadador o el pianista.
     ´
Cap´
   ıtulo 1

Principios Generales

                             “La principal raz´n de existir del matem´tico es re-
                                              o                       a
                             solver problemas, y por lo tanto en lo que realmente
                             consisten las matem´ticas es en problemas y solu-
                                                  a
                             ciones.”
                                                            Paul R. Halmos [14]



    En este cap´ ıtulo nos ocuparemos de los m´todos y principios generales
                                                e
que resultan utiles para la resoluci´n de problemas. Pero recordemos que la
              ´                     o
unica manera de aprender a resolver problemas es . . . resolviendo problemas!
´
Por lo tanto la lectura de este cap´
                                   ıtulo solamente ser´ util si se combina con
                                                      a´
la pr´ctica constante. Para quienes tengan poca experiencia es recomendable
     a
pasar r´pidamente por las p´ginas siguientes, para volver a ellas m´s tarde,
       a                      a                                       a
como referencia, mientras est´n trabajando en la resoluci´n de problemas
                                e                            o
concretos. Quienes se interesen por el estudio en profundidad de la habilidad
para resolver problemas pueden consultar [27].


1.1.    Resoluci´n de Problemas y Creatividad
                o
    Evidentemente la resoluci´n de problemas est´ estrechamente relaciona-
                              o                   a
da con la creatividad, que algunos definen precisamente como la habilidad
para generar nuevas ideas y solucionar todo tipo de problemas y desaf´   ıos.
    La especie humana es creativa por naturaleza. Todo ser humano nace
con un gran potencial para la creaci´n, pero mientras algunos lo aprovechan
                                     o
al m´ximo, otros casi no lo utilizan. Sin embargo la creatividad, al igual que
     a
4                                                        Principios Generales


cualquier otra habilidad humana, puede desarrollarse a trav´s de la pr´ctica
                                                               e          a
y el entrenamiento adecuado. Lamentablemente tambi´n puede atrofiarse, si
                                                         e
no se ejercita adecuadamente.
     El pensamiento creativo se ha dividido en divergente y convergente. El
primero consiste en la habilidad para pensar de manera original y elabo-
rar nuevas ideas, mientras que el segundo se relaciona con la capacidad
cr´ıtica y l´gica para evaluar alternativas y seleccionar la m´s apropiada.
            o                                                     a
Evidentemente ambos tipos de pensamiento juegan un rol fundamental en
la resoluci´n de problemas.
            o
     Tres aspectos de la creatividad han recibido mucha atenci´n: el proceso
                                                                  o
creativo, las caracter´
                      ısticas de la personalidad creativa, y las circunstancias
que posibilitan o favorecen el acto creativo. Como consecuencia de estos es-
tudios se han desarrollado t´cnicas y m´todos generales dirigidos a desarro-
                               e          e
llar el potencial creativo. En esta obra nos concentraremos en las t´cnicas
                                                                        e
y estrategias espec´ıficas que han demostrado ser m’s utiles para la resolu-
                                                          ´
ci´n de problemas matem´ticos. Sin embargo haremos a continuaci´n una
  o                          a                                          o
breve rese˜a de algunos de los m´todos m´s generales, remitiendo al lector
            n                       e        a
interesado a la bibliograf´ correspondiente.
                           ıa


1.1.1.   Invertir el problema
    Cada concepto tiene uno contrario y la oposici´n entre ellos genera una
                                                   o
tensi´n favorable al hecho creativo. Esta idea, que tiene profundas ra´
     o                                                                   ıces
tanto en la filosof´ oriental como en la occidental, se refleja en la sabidur´
                  ıa                                                       ıa
popular en aforismos tales como: “Para saber mandar hay que aprender
a obedecer” o “Para ser un buen orador hay que saber escuchar”. Como
ejemplo de esta t´cnica supongamos que deseamos dise˜ar un zapato que
                   e                                      n
sea muy c´modo. El problema inverso ser´ dise˜ar un zapato inc´modo.
           o                               ıa     n                   o
Pero el an´lisis de este problema nos llevar´ seguramente a descubrir los
           a                                 a
factores que causan incomodidad, y al evitarlos habremos dado un buen
paso hacia la soluci´n del problema original. Vea [38].
                     o


1.1.2.   Pensamiento lateral
     Consiste en explorar alternativas inusuales o incluso aparentemente ab-
surdas para resolver un problema. En otras palabras: evitar los caminos
trillados, intentar lo que nadie ha intentado, ensayar percepciones y puntos
de vista diferentes. Vea [5].
1.1 Resoluci´n de Problemas y Creatividad
            o                                                                5


1.1.3.   Principio de discontinuidad
    La rutina suprime los est´ ımulos necesarios para el acto creativo, por
lo tanto si experimenta un bloqueo temporal de su capacidad creadora in-
terrumpa su programa cotidiano de actividades y haga algo diferente a lo
acostumbrado. Vaya a dar un paseo por sitios que no conoce, ensaye una
nueva receta de cocina, escuche m´sica diferente a la que escucha habi-
                                     u
tualmente, lea un libro que no ten´ pensado leer, asista a alg´n tipo de
                                     ıa                         u
espect´culo diferente a sus favoritos.
      a

1.1.4.   Imitaci´n
                o
    La mayor parte de los grandes artistas comienzan imitando a sus maes-
tros. M´s a´n se ha llegado a afirmar, en parte en broma y en parte en serio,
        a u
que “la originalidad no es otra cosa que un plagio no detectado”. En cual-
quier caso es claro que la imitaci´n puede ser un primer paso v´lido hacia
                                  o                              a
la originalidad. En particular observe y no vacile en imitar las t´cnicas de
                                                                  e
resoluci´n de problemas empleadas con ´xito por sus compa˜eros, maestros
        o                               e                   n
o colegas.

1.1.5.   Tormenta de cerebros (Brainstorming)
    Es una t´cnica desarrollada en el mundo de la publicidad, en el cual el
             e
´xito depende de la generaci´n de nuevas y brillantes ideas. Para ello se
e                               o
re´ne un grupo de personas y se les invita a expresar todas las ideas que
  u
se les ocurran en relaci´n a un problema o tema planteado, sin importar lo
                         o
estrafalarias o rid´
                   ıculas que parezcan. La evaluaci´n y la cr´
                                                   o         ıtica se posponen,
esperando crear un clima estimulante que favorezca el surgimiento de algunas
ideas realmente utiles. La utilidad de esta t´cnica es dudosa fuera de ciertos
                  ´                          e
campos o situaciones muy espec´   ıficas.

1.1.6.   Mapas mentales
    Es una t´cnica desarrollada por Tony Buzan (vea [6] y [7]) que trata de
             e
representar en forma gr´fica el car´cter asociativo de la mente humana. Se
                        a          a
comienza con la idea principal ubicada en el centro de la hoja y alrededor
de ella se van colocando las ideas asociadas y sus respectivos v´
                                                                ınculos. Uti-
lizando diversos colores y s´
                            ımbolos esta t´cnica puede llegar a ser muy util
                                          e                              ´
para organizar las ideas que van surgiendo en torno a un problema.
6                                                        Principios Generales


1.1.7.   Programaci´n neuroling¨´
                   o           uıstica (PNL)
    Tambi´n conocida como “la ciencia de la experiencia subjetriva”, es un
          e
conjunto de t´cnicas muy desarrolladas a trav´s de las cuales se trata de
              e                                   e
caracterizar el contexto (f´
                           ısico, fisiol´gico, psicol´gico, ambiental, etc.) en
                                       o            o
el cual somos m´s creativos, para luego reproducirlo a voluntad. Los prac-
                 a
ticantes de la PNL han incluso “modelado” el comportamiento de algunos
personajes famosos, tales como Walt Disney, para tratar de aprovechar sus
modos y procedimientos m´s creativos. Vea [10] y [11].
                            a

1.1.8.   Factores afectivos
    La resoluci´n de problemas no es un asunto puramente intelectual. Las
               o
emociones, y en particular el deseo de resolver un problema, tienen tambi´n  e
una gran importancia. La incapacidad que manifiestan algunos alumnos para
resolver incluso el ejercicio m´s sencillo no es producto por lo general de una
                                 a
deficiencia intelectual, sino de una absoluta falta de inter´s y motivaci´n. A
                                                            e             o
veces no existe ni siquiera el deseo de comprender el problema, y por lo tanto
el mismo no es comprendido. El profesor que desee realmente ayudar a un
alumno con estas caracter´   ısticas deber´ ante todo despertarle su curiosidad
                                           a
dormida, motivarlo y transmitirle deseos de logro y superaci´n. o
    Algunas creencias negativas para el proceso creativo est´n asociadas a
                                                                a
una baja autoestima y pueden tener ra´ emocionales profundas. Por ejem-
                                           ıces
plo hay quienes enfrentados a un problema creen a priori que no podr´n       a
resolverlo, y que si lo intentan s´lo conseguir´n terminar con un dolor de
                                     o             a
cabeza. El maestro o profesor debe en estos casos apelar a todas sus dotes
y conocimientos como educador, aunque en casos extremos ser´ necesaria
                                                                    a
tambi´n la ayuda de un orientador o la de un psic´logo.
      e                                               o
    En el polo opuesto, alguien que tenga confianza en su propia capacidad
y crea que un problema es un desaf´ que vale la pena enfrentar y que re-
                                        ıo
solverlo le proporcionar´ una satisfacci´n intelectual al mismo tiempo que
                          a                  o
ser´ una experiencia valiosa para su formaci´n, estar´ en excelentes condi-
   a                                             o       a
ciones psicol´gicas para abordar el proceso resolutivo. Para profundizar en
             o
estos aspectos vea [4], [24], [25], [26].

1.1.9.   Bloqueos mentales
   James Adams, profesor de dise˜o en la Universidad de Stanford, centra su
                                  n
enfoque de la creatividad en la superaci´n de los bloqueos mentales, barreras
                                        o
1.2 La Creaci´n Matem´tica
             o       a                                                        7


que nos impiden percibir un problema en la forma correcta y encontrarle
soluci´n. En [1] analiza diferentes tipos de bloqueos y propone ejercicios
      o
para identificarlos y superarlos. Su clasificaci´n es la siguiente:
                                              o

       Bloqueos perceptivos: estereotipos, dificultad para aislar el proble-
       ma, delimitar demasiado el espacio de soluciones, imposibilidad de ver
       el problema desde varios puntos de vista, saturaci´n, no poder utilizar
                                                         o
       toda la informaci´n sensorial.
                        o

       Bloqueos emocionales: miedo a cometer errores, a arriesgar, a fra-
       casar; deseo de seguridad y orden; preferir juzgar ideas a concebirlas;
       inhabilidad para relajarse; falta de est´
                                               ımulo; entusiasmo excesivo; falta
       de control imaginativo.

       Bloqueos culturales: tab´es; el peso de la tradici´n; roles predeter-
                                u                        o
       minados asignados a la mujer y al hombre.

       Bloqueos ambientales: distracciones; falta de apoyo para llevar ade-
       lante una idea; falta de cooperaci´n entre colegas.
                                         o

       Bloqueos intelectuales: inhabilidad para seleccionar un lenguaje
       apropiado para el problema (verbal, matem´tico, visual); uso inade-
                                                     a
       cuado de las estrategias; falta de informaci´n o informaci´n incorrecta.
                                                   o             o

       Bloqueos expresivos: t´cnicas inadecuadas para registrar y expresar
                                e
       ideas (a los dem´s y a uno mismo)
                       a


1.2.      La Creaci´n Matem´tica
                   o       a
   Una de las reflexiones m´s profundas que se han hecho sobre la creativi-
                            a
dad en matem´tica es la realizada a principios de siglo por Henri Poincar´,
               a                                                         e
uno de los m´s grandes matem´ticos de su tiempo. En una conferencia pro-
             a                 a
nunciada ante la Sociedad Psicol´gica de Par´ [30] hizo interesant´
                                  o            ıs                    ısimas
revelaciones sobre sus propias experiencias como creador:

          “¿Qu´ es, de hecho, la creaci´n matem´tica? No consiste en
                e                      o         a
       hacer combinaciones nuevas con entes matem´ticos ya conoci-
                                                     a
       dos. Cualquiera podr´ hacerlo, pcro las combinaciones que se
                            ıa
       podr´ hacer as´ ser´ un n´mero limitado y en su mayor´
            ıan        ı    ıan      u                              ıa
       totalmente desprovistas de inter´s. Crear consiste precisamente
                                       e
8                                                           Principios Generales


       no en construir las combinaciones in´tiles, sino en construir las
                                             u
       que son utiles y que est´n en ´
                 ´                a   ınfima minor´ Crear es discernir,
                                                    ıa.
       es escoger. . . ”
           “A menudo, cuando se trabaja en un problema dif´      ıcil, no se
       consigue nada la primera vez que se comienza la tarea. Luego se
       toma un descanso m´s o menos largo y uno se sienta de nuevo
                               a
       ante la mesa. Durante la primera media hora se contin´a sin   u
       encontrar nada. Despu´s, de repente. la idea decisiva se presenta
                                 e
       ante la mente. . . ”
           “Hay que hacer otra observaci´n a prop´sito de las condicio-
                                          o          o
       nes de este trabajo inconsciente. Se trata de que tal trabajo no
       es posible, y en todo caso no es fecundo, si no est´ por una parte
                                                          a
       precedido y por otra seguido de un per´  ıodo de trabajo conscien-
       te. Estas inspiraciones s´bitas no se presentan . . . m´s que tr´s
                                   u                           a          a
       algunos d´ de esfuerzos voluntarios, aparentemente est´riles, en
                  ıas                                             e
       los que uno ha cre´ no hacer nada interesante, y piensa haber
                            ıdo
       tomado un camino falso totalmente. Estos esfuerzos no fueron,
       por tanto, tan est´riles como se pensaba. Pusieron en movimien-
                           e
       to la m´quina inconsciente y sin ellos ´sta no habr´ funcionado
               a                               e            ıa
       ni hubiera producido nada. . . ”

    Poincar´ esboza luego una teor´ del trabajo del yo subliminal, en la
            e                       ıa
cual atribuye un rol fundamental a la sensibilidad y el sentido est´tico del
                                                                    e
matem´tico en el proceso de selecci´n, durante el trabajo inconsciente, de
        a                            o
las combinaciones m´s significativas.
                     a
    Una conclusi´n pr´ctica: cuando un problema se resiste a nuestros mejo-
                  o   a
res esfuerzos, nos queda todav´ la posibilidad de dejarlo durante un tiempo,
                              ıa
descansar, dar un paseo, y volver a ´l m´s tarde. Sin embargo solamente
                                       e   a
aquellos problemas que nos han apasionado, manteni´ndonos en una con-
                                                        e
siderable tensi´n mental, son los que vuelven m´s tarde, transformados, a
                o                                 a
la mente consciente. La inspiraci´n o iluminaci´n s´bita, que los antiguos
                                  o              o u
consideraban un don divino, hay que merecerla.


1.3.      La metodolog´ de P´lya
                      ıa    o
     En 1945 el insigne matem´tico y educador George P´lya (1887–1985)
                              a                          o
public´ un libro que r´pidamente se convirtir´ en un cl´sico: How to solve
        o              a                     ıa        a
it [31]. En el mismo propone una metodolog´ en cuatro etapas para resolver
                                          ıa
1.3 La metodolog´ de P´lya
                ıa    o                                                     9


problemas. A cada etapa le asocia una serie de preguntas y sugerencias que
aplicadas adecuadamente ayudar´n a resolver el problema. Las cuatro etapas
                                 a
y las preguntas a ellas asociadas se detallan a continuaci´n:
                                                          o
Etapa I: Comprensi´n del problema.
                  o
     ¿Cu´l es la inc´gnita? ¿Cu´les son los datos? ¿Cual es la condici´n?
        a           o          a                                      o

     ¿Es la condici´n suficiente para determinar la inc´gnita? ¿Es insufi-
                   o                                  o
     ciente? ¿Redundante? ¿Contradictoria?
Etapa II: Concepci´n de un plan.
                  o
     ¿Se ha encontrado con un problema semejante? ¿Ha visto el mismo
     problema planteado en forma ligeramente diferente?

     ¿Conoce un problema relacionado con ´ste? ¿Conoce alg´n teorema
                                              e                  u
     que le pueda ser util? Mire atentamente la inc´gnita y trate de recordar
                      ´                            o
     un problema que le sea familiar y que tenga la misma inc´gnita o una
                                                                o
     inc´gnita similar.
        o

     He aqu´ un problema relacionado con el suyo y que se ha resuelto ya.
           ı
     ¿Podr´ utilizarlo? ¿Podr´ emplear su resultado? ¿Podr´ utilizar su
          ıa                 ıa                             ıa
     m´todo? ¿Podr´ utilizarlo introduciendo alg´n elemento auxiliar?
       e           ıa                           u

     ¿Podr´ enunciar el problema en otra forma? ¿Podr´ plantearlo en
          ıa                                            ıa
     forma diferente nuevamente? Refi´rase a las definiciones.
                                    e

     Si no puede resolver el problema propuesto, trate de resolver primero
     alg´n problema similar. ¿Podr´ imaginarse un problema an´logo un
        u                             ıa                              a
     tanto m´s accesible? ¿Un problema m´s general? ¿Un problema m´s
              a                                a                            a
     particular? ¿Un problema an´logo? ¿Puede resolver una parte del pro-
                                    a
     blema? Considere s´lo una parte de la condici´n; descarte la otra parte;
                        o                            o
     ¿en qu´ medida la inc´gnita queda ahora determinada? ¿en qu´ forma
            e               o                                          e
     puede variar? ¿Puede usted deducir alg´n elemento util de los datos?
                                                u            ´
     ¿Puede pensar en algunos otros datos apropiados para determinar la
     inc´gnita? ¿Puede cambiar la inc´gnita? ¿Puede cambiar la inc´gnita
        o                                o                               o
     o los datos, o ambos si es necesario, de tal forma que la nueva inc´gnita
                                                                         o
     y los nuevos datos est´n m´s cercanos entre s´
                            e     a                   ı?

     ¿Ha empleado todos los datos? ¿Ha empleado toda la condici´n? ¿Ha
                                                                 o
     considerado usted todas las nociones esenciales concernientes al pro-
     blema?
10                                                       Principios Generales


Etapa III: Ejecuci´n del plan.
                  o

      Al ejecutar el plan, compruebe cada uno de los pasos.

      ¿Puede ver claramente que el paso es correcto? ¿Puede demostrarlo?

Etapa IV. Visi´n retrospectiva.
              o

      ¿Puede usted verificar el resultado? ¿Puede verificar el razonamiento?

      ¿Puede obtener el resultado en forma diferente? ¿Puede verlo de golpe?
      ¿Puede emplear el resultado o el m´todo en alg´n otro problema?
                                         e            u

     La primera etapa es obviamente insoslayable: es imposible resolver un
problema del cual no se comprende el enunciado. Sin embargo en nuestra
pr´ctica como docentes hemos visto a muchos estudiantes lanzarse a efectuar
   a
operaciones y aplicar f´rmulas sin reflexionar siquiera un instante sobre lo
                         o
que se les pide. Por ejemplo si en el problema aparece una funci´n comienzan
                                                                  o
de inmediato a calcularle la derivada, independientemente de lo que diga el
enunciado. Si el problema se plantea en un examen y luego, comentando los
resultados, el profesor dice que el c´lculo de la derivada no se ped´ y m´s
                                       a                              ıa     a
a´n que el mismo era irrelevante para la soluci´n del problema, algunos le
 u                                                 o
responder´n: ¿o sea que no nos va a dar ning´n punto por haber calculado
           a                                     u
la derivada? Este tipo de respuesta revela una incomprensi´n absoluta de
                                                                o
lo que es un problema y plantea una situaci´n muy dif´ al profesor, quien
                                               o          ıcil
tendr´ que luchar contra vicios de pensamiento arraigados, adquiridos tal
      a
vez a lo largo de muchos a˜os.
                             n
     La segunda etapa es la m´s sutil y delicada, ya que no solamente est´ re-
                                a                                          a
lacionada con los conocimientos y la esfera de lo racional, sino tambi´n con
                                                                         e
la imaginaci´n y la creatividad. Observemos que las preguntas que P´lya
              o                                                            o
asocia a esta etapa est´n dirigidas a llevar el problema hacia un terreno co-
                         a
nocido. Con todo lo utiles que estas indicaciones son, sobre todo para el tipo
                      ´
de problemas que suele presentarse en los cursos ordinarios, dejan planteada
una interrogante: ¿qu´ hacer cuando no es posible relacionar el problema
                        e
con algo conocido? En este caso no hay recetas infalibles, hay que trabajar
duro y confiar en nuestra propia creatividad e inspiraci´n. o
     La tercera etapa es de car´cter m´s t´cnico. Si el plan est´ bien concebi-
                                 a       a e                    a
do, su realizaci´n es factible y poseemos los conocimientos y el entrenamiento
                o
necesarios, deber´ ser posible llevarlo a cabo sin contratiempos. Sin embar-
                  ıa
go por lo general en esta etapa se encontrar´n dificultades que nos obligar´n
                                              a                              a
1.4 El trabajo de Alan Schoenfeld                                           11


a regresar a la etapa anterior para realizar ajustes al plan o incluso para
modificarlo por completo. Este proceso puede reperirse varias veces.
    La cuarta etapa es muchas veces omitida, incluso por solucionistas exper-
tos. P´lya insiste mucho en su importancia, no solamente porque comprobar
      o
los pasos realizados y verificar su correcci´n nos puede ahorrar muchas sor-
                                           o
presas desagradables, sino porque la visi´n retrospectiva nos puede conducir
                                         o
a nuevos resultados que generalicen, ampl´ o fortalezcan el que acabamos
                                           ıen
de hallar.


1.4.     El trabajo de Alan Schoenfeld
    Si bien la mayor´ de los matem´ticos reconocen en las estrategias heur´
                    ıa              a                                       ıs-
ticas de P´lya los m´todos que ellos mismos utilizan habitualmente, no es
           o          e
tan f´cil para el que no tiene experiencia aplicarlas exitosamente. En otras
     a
palabras, dichas estrategias son m´s descriptivas que prescriptivas. Alan
                                     a
Schoenfeld (ver [34], [35], [36]) es uno de los que m´s han estudiado esta
                                                       a
problem´tica. En su an´lisis identifica los siguientes cuatro factores relevan-
         a              a
tes para la resoluci´n de problemas:
                    o

       Recursos cognitivos. Son nuestros conocimientos matem´ticos ge-
                                                                 a
       nerales, tanto de conceptos y resultados como de procedimientos (al-
       goritmos).

       Heur´ ıstica. Es el conjunto de estrategias y t´cnicas para resolver
                                                      e
       problemas que conocemos y estamos en capacidad de aplicar.

       Control o metacognici´n. Es la capacidad de utilizar lo que sabe-
                                o
       mos para lograr un objetivo.

       Creencias. Se refiere a aquellas creencias y opiniones relacionadas
       con la resoluci´n de problemas y que pueden afectarla favorable o
                      o
       desfavorablemente.

   La importancia del primer factor es obvia. Sin embargo se ha demostra-
do (ver [9]) que no es suficiente poseer un amplio bagaje de conocimientos
matem´ticos para ser un solucionista experto. Tambi´n es necesario dominar
       a                                               e
algunas t´cnicas y estrategias que nos ayuden a atacar el problema. En do-
         e
minios restringidos y bien delimitados, en los cuales los problemas a resolver
son m´s o menos rutinarios, se han desarrollado estrategias que pueden ser
      a
12                                                       Principios Generales


aplicadas con ´xito incluso por un computador, con resultados tan buenos o
                e
mejores que los obtenidos por los expertos humanos (estos son los famosos
sistemas expertos, producto de las investigaciones en inteligencia artificial
y ciencia cognitiva). Sin embargo para resolver problemas no rutinarios en
dominios ricos en contenido, como la matem´tica, se requiere algo m´s que
                                              a                       a
conocimientos y estrategias. Ese factor adicional es lo que llamamos con-
trol; act´a como una voz interior que nos dice qu´ ideas y estrategias (entre
          u                                      e
muchas alternativas posibles) nos conviene aplicar para el problema que te-
nemos entre manos, o bien si debemos abandonar un camino que no parece
arrojar resultados o por el contrario redoblar esfuerzos y perseverar en ´l.
                                                                           e
Los solucionistas inexpertos tienen evidentes deficiencias en este aspecto: se
apresuran a transitar el primer camino que se les ocurre y luego se mueven
en c´ırculos, cayendo una y otra vez en el mismo error.
    El ultimo factor puede influir tambi´n de manera importante en el pro-
        ´                                e
ceso de resoluci´n de problemas. Algunas creencias comunes, sobre todo
                  o
entre estudiantes de ense˜anza media, son las siguientes: “todo problema
                           n
se resuelve mediante alguna f´rmula”, “lo importante es el resultado y no
                                o
el procedimiento”, “la respuesta del libro no puede estar equivocada”. Este
tipo de creencias es un obst´culo para el desempe˜o de cualquier persona
                             a                      n
como solucionista.
    Schoenfeld elabor´ tambi´n una lista de las estrategias m´s utilizadas:
                      o       e                               a

 1. An´lisis.
      a

      a) Dibuje un diagrama siempre que sea posible.
      b) Examine casos especiales.
          1) Seleccione algunos valores especiales para ejemplificar el pro-
             blema e irse familiarizando con ´l.  e
          2) Examine casos l´     ımite para explorar el rango de posibilidades.
          3) Si hay un par´metro entero, dele sucesivamente los valores
                                a
             1, 2, . . . , m y vea si emerge alg´n patr´n inductivo.
                                                u        o
      c) Trate de simplificar el problema.
          1) Explotando la existencia de simetr´
                                               ıa.
          2) Usando argumentos del tipo “sin p´rdida de generalidad”.
                                               e

 2. Exploraci´n.
             o

      a) Considere problemas esencialmente equivalentes.
1.4 El trabajo de Alan Schoenfeld                                           13


          1) Reemplazando condiciones por otras equivalentes.
          2) Recombinando los elementos del problema de maneras dife-
             rentes.
          3) Introduciendo elementos auxiliares.
          4) Reformulando el problema:
                 Mediante un cambio de perspectiva o notaci´n.
                                                            o
                 Mediante argumentos por contradicci´n o contraposici´n.
                                                    o                o
                 Asumiendo que tenemos una soluci´n y determinando sus
                                                 o
                 propiedades.
      b) Considere un problema ligeramente modificado.
          1) Escoja submetas (tratando de satisfacer parcialmente las con-
             diciones).
          2) Relaje una condici´n y luego trate de reimponerla.
                               o
          3) Descomponga el dominio del problema y trabaje caso por
             caso.
      c) Considere problemas sustancialmente modificados.
          1) Construya un problema an´logo con menos variables.
                                        a
          2) Deje todas las variables fijas excepto una, para determinar
             su impacto.
          3) Trate de aprovechar cualquier problema relacionado que ten-
             ga forma, datos o conclusiones similares.

 3. Verificaci´n de la soluci´n.
             o              o

      a) ¿Pasa su soluci´n estas pruebas espec´
                        o                     ıficas?
          1) ¿Usa todos los datos pertinentes?
          2) ¿Est´ de acuerdo con estimaciones o predicciones razonables?
                 a
          3) ¿Soporta pruebas de simetr´ an´lisis dimensional y escala?
                                        ıa, a
      b) ¿Pasa estas pruebas generales?
          1)   ¿Puede   ser obtenida de manera diferente?
          2)   ¿Puede   ser sustanciada por casos especiales?
          3)   ¿Puede   ser reducida a resultados conocidos?
          4)   ¿Puede   utilizarse para generar alg´n resultado conocido?
                                                   u
Cap´
   ıtulo 2

Ejemplos sencillos

                             “Resolver un problema es hacer un descubrimiento.
                             Un gran problema significa un gran descubrimien-
                             to, pero hay una part´ıcula de descubrimiento en la
                             soluci´n de cualquier problema. El suyo puede ser
                                    o
                             modesto, pero si pone a prueba la curiosidad que
                             induce a poner en juego las facultades inventivas, y
                             si lo resuelve por medios propios, puede experimen-
                             tar la tensi´n y el encanto del descubrimiento y el
                                          o
                             goce del triunfo.”

                                                              George P´lya [31]
                                                                      o



    En este cap´ıtulo pondremos en pr´ctica los principios examinados en el
                                      a
cap´ıtulo anterior. Para ello hemos seleccionado varios problemas sencillos
y de f´cil soluci´n, de modo que nos podamos concentrar en el proceso de
       a         o
resoluci´n m´s que en el contenido de los mismos.
         o    a


2.1.                 ´
        Aritm´tica y Algebra
             e
    Algunos de los problemas m´s antiguos que se conocen son de tipo
                                  a
aritm´tico. Es t´
      e         ıpico que se pida hallar una cantidad determinada por cier-
tas condiciones, o bien efectuar un reparto cumpliendo ciertos requisitos.
Los siguientes problemas pertenecen a esta categor´ ıa.
Problema 2.1. Diofanto fue un notable matem´tico griego que desarroll´ su
                                                 a                       o
actividad en Alejandr´ en el siglo III A.C. y del cual se conservan muy pocos
                     ıa
e       ´
2.1 Aritm´tica y Algebra                                                      15


datos biogr´ficos. Sin embargo se dice que su epitafio conten´ la siguiente
           a                                               ıa
inscripci´n:
         o
          Caminante: aqu´ yacen los restos de Diofanto. Y los n´me-
                          ı                                       u
      ros pueden mostrar cu´n larga fue su vida, cuya sexta parte
                              a
      constituy´ su hermosa infancia. Hab´ transcurrido adem´s una
                o                          ıa                  a
      duod´cima parte cuando sus mejillas se cubrieron de vello. Lue-
            e
      go de una s´ptima parte se cas´, y transcurrido un quinquenio
                    e                 o
      le hizo dichoso el nacimiento de su primog´nito, cuya existencia
                                                e
      dur´ tan s´lo la mitad de la de su padre. Luego de cuatro a˜os
          o       o                                                 n
      buscando consuelo en la ciencia de los n´meros, descendi´ Dio-
                                              u                 o
      fanto a la sepultura.
¿Qu´ edad alcanz´ Diofanto? ¿A qu´ edad se cas´? ¿Cu´ntos a˜os vivi´ su
    e           o                e            o     a      n       o
hijo?
Soluci´n. Veamos si comprendemos bien el problema. ¿Cu´l es la inc´gnita?
       o                                                      a         o
El n´mero de a˜os que vivi´ Diofanto (las preguntas restantes se responden
     u          n            o
f´cilmente conociendo la respuesta a la primera). ¿Cu´les son los datos? Una
 a                                                      a
serie de informaciones sobre las etapas sucesivas de su vida, desde su infan-
cia hasta su muerte. Ahora debemos concebir un plan. ¿Se ha encontrado
con un problema semejante? Es de esperar que s´ ya que la mayor´ de los
                                                    ı,                 ıa
problemas resolubles por m´todos algebraicos elementales son semejantes.
                              e
El plan general consiste en escribir ecuaciones que reflejen las condiciones
planteadas, resolver el sistema resultante y finalmente interpretar las solu-
ciones obtenidas en el contexto original del problema. Llamemos x al n´mero
                                                                          u
de a˜os vividos por Diofanto. Esta cantidad debe ser igual a la suma de las
     n
duraciones de las etapas de su vida, a saber: su infancia (x/6), la duod´cima
                                                                          e
parte transcurrida hasta que le sali´ barba (x/12), los a˜os transcurridos
                                       o                      n
hasta que contrajo matrimonio (x/7), los a˜os transcurridos hasta que na-
                                              n
ci´ su primog´nito (5), los a˜os que ´ste vivi´ (x/2) y los 4 a˜os que Diofanto
  o           e              n       e        o                n
le sobrevivi´. Por lo tanto escribimos:
            o
                              x    x     x        x
                         x= +        + +5+ +4                               (2.1)
                              6 12 7              2
Agrupando t´rminos semejantes resulta:
              e
                              1   1  1 1
                       (1 −     −   − − )x = 5 + 4
                              6 12 7 2
y simplificando queda
                                    3
                                      x = 9.
                                   28
16                                                          Ejemplos sencillos


Por lo tanto x = 28 × 9/3 = 84. Verifiquemos el resultado:

        84 84 84       84
          +    +   +5+    + 4 = 14 + 7 + 12 + 5 + 42 + 4 = 84
        6   12   7     2
Diofanto se cas´ cuando contaba 84/6 + 84/12 + 84/7 = 33 a˜os, y su hijo
               o                                          n
vivi´ 84/2 = 42 a˜os.
    o            n

    Los documentos matem´ticos m´s antiguos que se conservan son dos
                           a        a
rollos de papiro egipcios que datan aproximadamente de la XII dinast´   ıa
(2078 a 1788 A.C.). Uno de ellos, conocido como el papiro Rhind, consta de
unos 85 problemas y ejemplos pr´cticos. El siguiente es uno de ellos:
                                 a

Problema 2.2. Dividir cien panes entre cinco hombres, de modo que las
porciones que reciban est´n en progresi´n aritm´tica y que la s´ptima parte
                         e             o       e               e
de la suma de las tres mayores sea igual a la suma de las dos porciones
menores.

Soluci´n. Asegur´monos de comprender bien el problema. ¿Qu´ se nos pide?
       o           e                                              e
Dividir cien panes entre cinco hombres, de modo que se cumplan ciertas con-
diciones. ¿Cu´les son los datos? El n´mero total de panes (100), la cantidad
              a                        u
de porciones (5) y las condiciones que debe cumplir el reparto. ¿Cu´les son
                                                                      a
las inc´gnitas? Obviamente, la cantidad de panes que le corresponder´ a ca-
       o                                                                a
da uno. ¿Comprendemos la condici´n? En primer lugar las porciones deben
                                     o
estar en progresi´n aritm´tica; esto significa que si escribimos las porciones
                  o        e
en orden creciente de magnitud, la diferencia de cada una de ellas con la
siguiente es constante. En otras palabras, si llamamos x a la menor de las
porciones y r a la diferencia com´n o raz´n de la progresi´n, entonces las
                                   u       o                  o
cinco porciones deber´n ser x, x + r, x + 2r, x + 3r y x + 4r. Utilizando esta
                       a
notaci´n podemos describir la ultima condici´n del problema mediante una
       o                        ´             o
ecuaci´n:
       o
                 (x + 2r) + (x + 3r) + (x + 4r)
                                                = x + (x + r)             (2.2)
                               7
¿Es la condici´n suficiente para determinar la inc´gnita? ¿Es insuficiente?
               o                                    o
Estas preguntas vienen muy bien en este momento, ya que nos hacen ob-
servar que tenemos dos inc´gnitas x y r pero una sola ecuaci´n. En general
                             o                                  o
(pero por supuesto hay excepciones) esto significa que el problema es inde-
terminado, es decir que en vez de una unica soluci´n admite varias, tal vez
                                         ´          o
hasta un n´mero infinito de ellas. Pero otra posibilidad a tener en cuenta
            u
es que no tengamos suficientes ecuaciones sencillamente por haber pasado
e       ´
2.1 Aritm´tica y Algebra                                                    17


por alto alg´n dato o condici´n del problema. Recordemos las preguntas
             u                 o
de P´lya: ¿Ha empleado todos los datos?, ¿Ha empleado toda la condici´n?
     o                                                                    o
Bueno, leyendo una vez m´s el enunciado del problema vemos que no hemos
                           a
utilizado el hecho de que los panes a dividir son cien. Este dato nos permite
escribir otra ecuaci´n:
                    o

             x + (x + r) + (x + 2r) + (x + 3r) + (x + 4r) = 100           (2.3)

Bien, ya tenemos dos ecuaciones y dos inc´gnitas. El plan a seguir es simple:
                                          o
resolver el sistema. Para ello simplificamos primero las ecuaciones 2.2 y 2.3
hasta obtener

                              11x − 2r = 0                                (2.4)
                                 x + 2r = 20                              (2.5)

de donde resulta x = 5/3 y r = 55/6. Las cinco porciones ser´n entonces:
                                                                 a
5/3 = 1 2 , 5/3 + 55/6 = 65/6 = 10 5 , 65/6 + 55/6 = 20, 20 + 55/6 = 175/6 =
        3                          6
29 1 y finalmente 175/6 + 55/6 = 115/3 = 38 1 .
   6                                           3
Visi´n retrospectiva: ¿Puede usted verificar el resultado? Esto es f´cil: 5/3 +
    o                                                                a
65/6 + 20 + 175/6 + 115/3 = 100 y 65/6 − 5/3 = 20 − 65/6 = 175/6 − 20 =
115/3 − 175/6 = 55/6. ¿Puede obtener el resultado en forma diferente?
Bueno, si se tiene cierta experiencia resolviendo problemas con progresiones
aritm´ticas se observa que muchas veces resulta m´s c´modo representar la
      e                                               a o
progresi´n de manera sim´trica, alrededor de un t´rmino central. En nuestro
          o                e                        e
caso, si llamamos z al t´rmino central y r a la raz´n, los cinco t´rminos ser´n
                        e                          o              e          a
z − 2r, z − r, z, z + r y z + 2r. Ahora la condici´n de que las partes suman
                                                   o
cien se escribe as´ı:

             (z − 2r + +(z − r) + z + (z + r) + (z + 2r) = 100

que se reduce a 5z = 100 y por tanto z = 20. La otra condici´n es
                                                            o
              20 + (20 + r) + (20 + 2r)
                                        = (20 − 2r) + (20 − r)
                          7
que luego de simplificar nos da 60 + 3r = 7(40 − 3r), de donde podemos
despejar r = (280 − 60)/24 = 55/6. Obtenemos por supuesto la misma
soluci´n que antes, pero el procedimiento luce m´s limpio y elegante: en lugar
      o                                         a
de resolver un sistema de dos ecuaciones con dos inc´gnitas s´lo tenemos que
                                                     o        o
resolver un par de ecuaciones de primer grado. Esto se debe a que la simetr´ıa
hace que se cancelen los t´rminos con r en la primera ecuaci´n.
                           e                                   o
18                                                            Ejemplos sencillos


Problema 2.3. Tres recipientes contienen agua. Si se vierte 1/3 del conte-
nido del primer recipiente en el segundo, y a continuaci´n 1/4 del contenido
                                                        o
del segundo en el tercero, y por ultimo 1/10 del contenido del tercero en el
                                  ´
primero, entonces cada recipiente queda con 9 litros de agua. ¿Qu´ cantidad
                                                                  e
de agua hab´ originalmente en cada recipiente?
            ıa

Soluci´n. Este problema puede tratarse en principio con el mismo m´todo
       o                                                                    e
que los anteiores: si llamamos x, y, z a los contenidos iniciales de los recipien-
tes es posible escribir unas ecuaciones que reflejen las condiciones del pro-
blema. Por ejemplo, despu´s de la primera operaci´n el contenido del primer
                            e                        o
recipiente ser´ (2/3)x y el del segundo y + x/3. Luego de la segunda opera-
              a
ci´n el contenido del segundo recipiente ser´ (3/4)(y + x/3) = x/4 + (3/4)y
  o                                            a
y el del tercero z + (1/4)(y + x/3) = x/12 + y/4 + z. Luego de la tercera
operaci´n el contenido del tercer recipiente ser´ (9/10)(x/12 + y/4 + z) y el
        o                                         a
del primero (2/3)x + (1/10)(x/12 + y/4 + z). Igualando ahora el contenido
final de cada recipiente con 9 obtenemos un sistema de tres ecuaciones con
tres inc´gnitas, cuya soluci´n es la respuesta buscada. Los detalles se los
         o                    o
dejamos al lector como ejercicio.
Visi´n retrospectiva: No cabe duda de que el m´todo anterior, aunque in-
     o                                               e
falible, es bastante aburrido y proclive a errores num´ricos. ¿No habr´ otra
                                                          e               a
forma de proceder m´s apropiada para este tipo de problema? S´ la hay,
                        a                                               ı
y consiste en sustituir el an´lisis hacia adelante que realizamos, partiendo
                              a
de la configuraci´n inicial y estudiando la evoluci´n del contenido de los
                   o                                    o
recipientes con cada operaci´n, por un an´lisis retrospectivo. Este tipo de
                               o              a
an´lisis consiste en partir de la configuraci´n final y estudiar c´mo se lleg´ a
   a                                         o                    o           o
ella. En nuestro caso los tres recipientes finalizan con 9 litros, y la ultima
                                                                          ´
operaci´n consisti´ en trasvasar 1/10 del contenido del tercer recipiente al
         o          o
primero. Pero si el tercer recipiente, luego de perder la d´cima parte de su
                                                              e
contenido, qued´ con 9 litros, es obvio que deb´ contener diez litros. Y el
                 o                                  ıa
primero, como qued´ con 9 luego de ganar un litro, antes conten´ 8 litros.
                      o                                              ıa
En otras palabras, despu´s de la segunda operaci´n y antes de la tercera el
                           e                          o
contenido de los recipientes era 8, 9 y 10 litros, en ese orden. Del mismo mo-
do se ve que antes de la segunda operaci´n el segundo recipiente conten´ 12
                                          o                                 ıa
litros, para poder quedar en 9 al perder la cuarta parte de su contenido. Y el
tercero, por consiguiente, ten´ 7 litros. Los contenidos antes de la segunda
                                ıa
operaci´n eran entonces 8, 12 y 7. Razonando de igual forma llegamos a que
         o
inicialmente los recipientes conten´ 12, 8 y 10 litros de agua. Este an´lisis
                                     ıan                                    a
retrospectivo se resume en la siguiente tabla:
2.2 Combinatoria                                                               19


                                  1◦    2◦   3◦
                                   9     9    9
                                   8     9   10
                                   8    12    7
                                  12     8   10




2.2.      Combinatoria
    Hay una clase importante de problemas en los cuales tenemos que contar
o enumerar configuraciones resultantes de combinar, de alguna manera, un
n´mero finito de elementos. La rama de la matem´tica que se ocupa de esto
 u                                               a
se conoce como combinatoria. Los siguientes son algunos ejemplos sencillos.
Problema 2.4. Un cubo s´lido de madera de lado 20 cm se pinta de rojo.
                          o
Luego con una sierra se hacen cortes paralelos a las caras, de cent´
                                                                   ımetro en
    ımetro, hasta obtener 203 = 8000 cubitos de lado 1 cm. ¿Cu´ntos de
cent´                                                               a
esos cubitos tendr´n al menos una cara pintada de rojo?
                  a
Soluci´n. El problema es de f´cil comprensi´n. El primer plan que se nos
         o                       a             o
ocurre es sencillamente contar los cubitos pintados. Por ejemplo: en cada
cara del cubo hay 202 = 400 cubitos pintados, por lo tanto en total ser´n  a
. . . ¿400×6? No, porque estar´ıamos contando m´s de una vez los cubitos que
                                                  a
est´n en los v´rtices y aristas del cubo. Pero al menos esto nos da una pista
     a         e
para mejorar el plan (y una cota superior: el n´mero de cubitos pintados
                                                    u
debe ser menor que 2400). Contemos entonces por separado los diferentes
tipos de cubitos pintados:
       Los correspondientes a los v´rtices del cubo, que tienen tres caras
                                     e
       pintadas y son ocho en total.
       Los correspondientes a las aristas del cubo, exclu´ ıdos los v´rtices (tie-
                                                                     e
       nen exactamente dos caras pintadas). Cada arista tiene contacto con
       20 cubitos, pero dos de ellos son v´rtices (que ya contamos aparte) por
                                          e
       lo cual nos quedan 18. Como el cubo tiene 12 aristas, el n´mero total
                                                                     u
       es 18 × 12 = 216.
       Los cubitos con exactamente una cara pintada. En cada cara del cubo,
       las caras pintadas de estos cubitos forman un cuadrado de 18 × 18, por
       lo tanto en total ser´n 18 × 18 × 6 = 1944.
                            a
20                                                          Ejemplos sencillos


Por consiguiente la respuesta es 8 + 216 + 1944 = 2168.
Visi´n retrospectiva: ¿Podemos obtener el resultado en forma diferente? Una
    o
primera alternativa es partir de nuestro primer resultado err´neo, 2400, y
                                                              o
efectuar las correcciones necesarias. Como los cubos de los v´rtices se con-
                                                             e
taron tres veces cada uno, restemos 8 × 2 = 16. Y como los de las aristas se
contaron dos veces, restemos 216. El resultado ser´ 2400 − 16 − 216 = 2168.
                                                  a
Otra idea (posiblemente la m´s elegante) se obtiene invirtiendo el proble-
                               a
ma. Contemos los cubitos que no tienen ninguna cara pintada. Es claro
que estos cubitos forman un cubo interior al primero, de lado 18. Por lo
tanto son 183 = 5832. Los que tiene al menos una cara pintada se pueden
obtener ahora restando esta ultima cantidad del total de cubitos, a saber
                              ´
203 − 183 = 8000 − 5832 = 2168.

Problema 2.5. En cada una de las 64 casillas de un tablero de ajedrez hay
un grano de az´car. Una hormiga comienza en un v´rtice del tablero, come
                u                                      e
el az´car, y se traslada a una casilla adyacente, desplaz´ndose en direcci´n
     u                                                     a                o
horizontal o vertical (pero nunca en diagonal). Contin´a de este modo hasta
                                                         u
acabar con todo el az´car, y sin pasar dos veces por una misma casilla. ¿Es
                      u
posible que su trayecto finalice en el v´rtice diagonalmente opuesto al inicial?
                                       e

Soluci´n. Este problema es de naturaleza diferente a los anteriores. No se
       o
nos pide calcular nada, por lo cual muchos pensar´n que no es un verdadero
                                                    a
problema de matem´tica. Sin embargo, si hacemos abstracci´n de la hormiga
                       a                                     o
y el az´car (que obviamente se han incluido para hacer m´s atractivo el
        u                                                      a
enunciado) vemos que el problema trata de la existencia de trayectorias con
ciertas caracter´ısticas geom´tricas.
                               e
    Por alguna raz´n, la mayor´ de las personas a quienes les he planteado
                     o            ıa
este problema contestan de inmediato que s´ Cuando les pido que dibujen
                                               ı.
en la pizarra la trayectoria, demuestran que no han comprendido cabalmente
el enunciado: trazan l´  ıneas diagonales, pasan m´s de una vez por la misma
                                                  a
casilla o simplemente finalizan en un v´rtice que no es el opuesto al inicial,
                                           e
y a´n as´ creen haber solucionado el problema. Cuando por fin comprenden
    u     ı
las condiciones, luego de dos o tres intentos fallidos cambian s´bitamen-
                                                                   u
te de posici´n y contestan que es imposible. Ahora bien, es claro que una
             o
respuesta afirmativa queda suficientemente justificada con s´lo exhibir una
                                                              o
trayectoria que cumpla las condiciones pedidas. ¿Pero c´mo podemos jus-
                                                           o
tificar una respuesta negativa? Es muy importante comprender la enorme
diferencia que existe entre las afirmaciones “no puedo hallar ninguna solu-
ci´n” y “no existe ninguna soluci´n”. Para poder afirmar esto ultimo hay
  o                                  o                            ´
2.3 Geometr´
           ıa                                                               21


b´sicamente dos maneras de proceder. Una de ellas consiste en dibujar todas
  a
las trayectorias posibles que parten de un v´rtice y recorren todo el tablero,
                                              e
desplaz´ndose en direcci´n horizontal o vertical y sin pasar dos veces por
        a                  o
ninguna casilla. Una vez hecho esto podemos examinar las trayectorias y
verificar que ninguna finaliza en el v´rtice opuesto al inicial. Un inconve-
                                        e
niente de este procedimiento es que resulta muy lento y engorroso para un
ser humano, aunque ser´ factible realizarlo con ayuda del computador. Otro
                         ıa
inconveniente es que si se nos ocurre generalizar el problema para tableros
m´s grandes r´pidamente el problema se vuelve inmanejable, incluso para
   a            a
el computador. M´s a´n, si queremos una respuesta general, para tableros
                    a u
de n × n, este procedimiento resulta completamente in´til.
                                                         u
     La segunda manera de proceder es demostrar que no existe trayectoria
alguna que cumpla las condiciones exigidas. Para esto resulta util el hecho
                                                                  ´
de que las casillas de un tablero de ajedrez est´n pintadas de dos colores,
                                                  a
digamos blanco y negro, en forma alternada. La observaci´n clave es que
                                                               o
cada movimiento unitario en direcci´n horizontal o vertical nos lleva de una
                                      o
casilla a otra de diferente color. Ahora bien, como el tablero tiene 8 × 8 = 64
casillas, comenzando en cualquiera de ellas se requieren 63 movimientos para
recorrerlas todas. Pero es claro que despu´s de 1, 3, 5 o cualquier n´mero
                                             e                           u
impar de movimientos estaremos en una casilla de color diferente a la inicial.
Esto demuestra que la respuesta al problema que nos ocupa es negativa, ya
que un v´rtice y el opuesto son del mismo color.
           e
Visi´n retrospectiva: Una generalizaci´n obvia de este problema consiste en
     o                                 o
considerar tableros de n × n, para cualquier entero positivo n. Es claro que
si n es par entonces la respuesta es negativa, por el mismo argumento usado
para el caso 8 × 8. En cambio si n es impar el argumento no se aplica. De
hecho es f´cil ver que la respuesta es afirmativa. Otras generalizaciones que
           a
se resuelven con el mismo m´todo: especificar dos casillas cualesquiera como
                             e
inicio y fin de la trayectoria; cambiar el tipo de movimiento b´sico, usando
                                                               a
por ejemplo saltos de caballo; plantear el problema en tres dimensiones, por
ejemplo en un cubo.

   El lector interesado en estos temas puede consultar [29].


2.3.     Geometr´
                ıa
    La otra clase importante de problemas que encontramos en la matem´ti-
                                                                     a
ca elemental son los de geometr´ El lector interesado en este tema puede
                                 ıa.
22                                                          Ejemplos sencillos


consultar [12].
    Hay una gran variedad de problemas geom´tricos: problemas de construc-
                                              e
ci´n, de c´lculo, de demostraci´n, etc. El siguiente es un ejemplo sencillo.
  o       a                    o

Problema 2.6. Los lados del tri´ngulo ABC miden AB = 26cm, BC =
                                   a
17cm y CA = 19cm. Las bisectrices de los ´ngulos de v´rtices B y C se
                                              a           e
cortan en el punto I. Por I se traza una paralela a BC que corta a los lados
AB y BC en los puntos M y N respectivamente. Calcule el per´     ımetro del
tri´ngulo AM N .
   a

Soluci´n. La primera de las estrategias que Schoenfeld coloca en su lista
       o
es hacer un diagrama, toda vez que sea posible. Si bien esta recomendaci´n o
se aplica a todo tipo de problemas, es casi insoslayable si el problema es
de car´cter geom´trico. Muchas veces el enunciado de estos problemas va
       a           e
acompa˜ado de un dibujo, pero otras veces (como en este caso) no es as´
         n                                                                   ı,
y hacerlo es la primera tarea que debemos realizar. Tal vez usted haya o´ ıdo
frases tales como “un dibujo no constituye demostraci´n”, “razonar en base
                                                      o
a un dibujo puede conducir a errores”, etc. Todo eso es cierto, sin embargo
un dibujo nos ayuda en primer lugar a comprender el problema. Adem´s       a
estimular´ nuestra imaginaci´n y es posibleque nos sugiera alg´n plan para
           a                   o                               u
hallar la soluci´n. Si tiene a mano instrumentos geom´tricos uselos; sin em-
                o                                     e      ´
bargo incluso un bosquejo aproximado suele ser de mucha ayuda (¡H´galo a
antes de seguir leyendo!).




Hay muchas maneras de resolver este problema. El que tenga afici´n a los
                                                                     o
c´lculos complicados podr´ por ejemplo comenzar por hallar el ´rea del
 a                         ıa                                        a
tri´ngulo ABC (usando la f´rmula de Heron). Dividiendo el ´rea entre el
   a                          o                                 a
semiper´ımetro se obtiene el radio de la circunferencia inscripta, es decir la
distancia de I a los lados del tri´ngulo ABC. Con estos datos es posible
                                  a
2.3 Geometr´
           ıa                                                             23


calcular, por proporcionalidad, las longitudes de AM , M N y AN . Sin em-
bargo esto es bastante engorroso. ¿No habr´ una manera m´s sencilla? Si
                                              a                  a
miramos el dibujo detenidamente, buscando alguna relaci´n interesante, ob-
                                                            o
servaremos (sobre todo si el dibujo est´ bien hecho) que los tri´ngulos BM I
                                       a                         a
y CN I parecen is´sceles. Si esto fuese cierto la soluci´n ser´ inmediata, ya
                  o                                     o     ıa
que de las igualdades M I = M B y IN = N C se obtiene:

AM + M N + AN       = AM + M I + IN + AN = AM + M B + AN + N C
                    = AB + AC = 26 + 19 = 45.

    Ahora bien, ¿podremos probar que los tri´ngulos BM I y CN I son is´sce-
                                            a                         o
les? Para probar por ejemplo que BM I es is´sceles es suficiente probar que
                                             o
los ´ngulos ∠M BI y ∠M IB son iguales. Pero sabemos que M N es paralela
    a
a BC, por lo tanto ∠M IB = ∠IBC ya que son ´ngulos alternos internos.
                                                 a
Pero BI es la bisectriz de ∠ABC, por lo tanto ∠M BI = ∠IBC y hemos
completado la demostraci´n (por supuesto que para el tri´ngulo CN I se
                          o                                a
razona de modo an´logo).
                   a
Visi´n retrospectiva: Si revisamos los datos del problema vemos que hay
     o
uno de ellos que no fue utilizado: la longitud del lado BC. En realidad para
cualquier tri´ngulo con AB = 26 cm y CA = 19 la soluci´n ser´ la misma,
             a                                            o      ıa
26 + 19 = 45. ¿Y si variamos AB y CA? Bueno, es f´cil ver que la respuesta
                                                      a
ser´ siempre AB + CA. En otras palabras, los valores 26 y 19 no juegan
   a
ning´n papel especial, y mucho menos BC = 17. Estos datos en vez de ayu-
     u
dar a resolver el problema m´s bien estorban, dirigiendo nuestra atenci´n
                               a                                          o
hacia detalles sin importancia. Son elementos distractores , que aumentan la
dificultad del problema suministrando m´s informaci´n que la estrictamente
                                          a            o
necesaria para resolverlo. Para aclarar mejor este punto supongamos que el
enunciado del problema hubiese sido:

     En un tri´ngulo ABC las bisectrices de los ´ngulos de v´rtices B
               a                                  a            e
     y C se cortan en el punto I. Por I se traza una paralela a BC que
     corta a los lados AB y BC en los puntos M y N respectivamente.
     Calcule el per´ımetro del tri´ngulo AM N en funci´n de los lados
                                  a                     o
     AB y AC.

Este problema, a pesar de ser m´s general, es probablemente m´s f´cil de
                                a                              a a
resolver ya que nuestra atenci´n se enfocar´ directamente hacia los lados
                              o            a
AB y AC. Este es el sentido de la recomendaci´n de P´lya: “eonsidere un
                                               o      o
problema m´s general”, la cual parece parad´jica ya que un problema m´s
            a                               o                          a
24                                                          Ejemplos sencillos


general deber´ ser por l´gica m´s dif´ Sin embargo una abstracci´n ade-
              ıa         o       a    ıcil.                           o
cuada, al eliminar la hojarasca innecesaria, puede permitirnos ver el camino
con m´s claridad. Ahora bien, ¿es posible detectar y evitar el efecto de es-
       a
tos elementos distractores? Es bastante dif´ıcil, ya que a priori no podemos
saber cu´les datos son esenciales y cu´les superfluos para resolver un pro-
         a                             a
blema. Sin embargo es razonable desconfiar de los datos que parecen muy
particulares para la naturaleza del problema. Pero hay que tener cuidado,
ya que hay propiedades que s´ dependen de valores muy particulares de los
                              ı
datos (esto es muy com´n en problemas de aritm´tica, por ejemplo).
                        u                           e

    Muchos problemas no se pueden clasificar de manera clara dentro de una
rama de la matem´tica, sino que se encuentran en la frontera entre dos o
                    a
m´s de ellas. El siguiente, por ejemplo, pertenece tanto a la geometr´ como
  a                                                                  ıa
a la combinatoria.

Problema 2.7. ¿En cu´ntas regiones queda dividido el plano por 6 rectas
                         a
en posici´n gen´rica (es decir tales que no haya dos de ellas paralelas ni tres
         o     e
concurrentes en un punto)?

Soluci´n. Evidentemente una recta divide el plano en dos regiones, y dos
       o
rectas no paralelas lo dividen en cuatro. Pero ya para tres rectas el problema
comienza a complicarse. Si trazamos unos cuantos diagramas veremos que
la tercera recta atraviesa siempre a tres de las cuatro regiones determinadas
por las dos primeras, pero no a la cuarta, y por lo tanto la respuesta para
tres rectas parece ser siete. ¿Pero podemos estar seguros de esto? ¿Y qu´ pa-
                                                                          e
sar´ cuando tracemos la cuarta, la quinta y la sexta recta? Lamentablemente
    a
los dibujos se complican demasiado, algunas rectas se cortan fuera de la ho-
ja y no es f´cil contar las regiones sin equivocarnos. Adem´s pareciera que
            a                                                  a
la respuesta depende de como dibujemos las rectas. Volvamos entonces al
principio. ¿Podr´ imaginarse un problema an´logo un tanto m´s accesible?
                 ıa                             a                   a
Bueno, en vez de disminu´ el n´mero de rectas podemos disminu´ la di-
                             ır    u                                   ır
mensi´n, es decir considerar en cu´ntas regiones queda dividida una recta
       o                             a
por cierto n´mero de puntos. Este problema s´ es f´cil, n puntos dividen a
             u                                   ı    a
la recta en n + 1 regiones (a saber n − 1 segmentos y 2 semirrectas). ¿Y no
podemos aprovechar este resultado para el problema en el plano? Veamos,
si ya hemos trazado n − 1 rectas entonces al trazar la n-sima ´sta cortar´ a
                                                                  e         a
las anteriores en n − 1 puntos diferentes (por la hi´tesis de genericidad). Por
                                                    o
lo tanto la n-sima recta quedar´ dividida en n partes por esos puntos de
                                   a
intersecci´n. Pero es claro que cada una de esas partes estar´ contenida por
          o                                                     a
2.3 Geometr´
           ıa                                                              25


completo en una regi´n de las determinadas por las primeras n − 1 rectas,
                      o
regi´n que quedar´ dividida en dos por la n-sima recta. Por lo tanto hemos
    o              a
descubierto que al trazar la n-sima recta el n´mero de regiones aumenta en
                                               u
n unidades. Apliquemos ahora este resultado desde el comienzo y de mane-
ra sucesiva. Inicialmente hay una sola regi´n: el plano. Al trazar la primera
                                            o
recta el n´mero de regiones aumenta en una unidad, y tendremos 1 + 1 = 2
          u
regiones. Al trazar la segunda recta el n´mero de regiones aumenta en dos
                                          u
unidades, y tendremos 2 + 2 = 4 regiones. Al trazar la tercera recta el n´me-
                                                                         u
ro de regiones aumenta en tres unidades, y tendremos 4 + 3 = 7 regiones.
Hasta aqu´ los resultados concuerdan con lo que ya sab´
           ı                                             ıamos. Ahora resulta
f´cil continuar: para cuatro rectas son 7 + 4 = 11 regiones, para cinco rectas
 a
son 11 + 5 = 16 regiones, para seis rectas son 16 + 6 = 22 regiones.
Visi´n retrospectiva: Resulta natural preguntarse cu´l ser´ el n´mero de re-
    o                                               a     a     u
giones en que queda dividido el plano por un n´mero n cualquiera de rectas
                                              u
en posici´n gen´rica. Recordando que la suma de los enteros desde 1 hasta
         o      e
n es n(n + 1)/2 es f´cil obtener
                    a

        1 + 1 + 2 + 3 + · · · + n = 1 + n(n + 1)/2 = (n2 + n + 2)/2

Hay otras generalizaciones y problemas similares a los cuales se puede aplicar
el mismo m´todo.
           e
Cap´
   ıtulo 3

Algunas Estrategias B´sicas
                     a

    En este cap´ıtulo se enuncian algunas estrategias b´sicas y se ilustra su
                                                       a
aplicaci´n a la soluci´n de varios problemas, muchos de ellos tomados de
        o              o
competencias matem´ticas internacionales.
                      a


3.1.     Figuras y diagramas
    El proverbio una figura vale m´s que mil palabras tiene plena validez en
                                 a
la resoluci´n de problemas matem´ticos. Por eso nuestra primera estrategia
           o                     a
es:

Estrategia 1. Dibuje una figura o un diagrama siempre que sea
posible.

    La importancia de este principio es obvia cuando se trata de resolver un
problema de geometr´ Pero hay muchos problemas que, sin ser de geome-
                     ıa.
tr´ admiten una interpretaci´n geom´trica, lo cual ampl´ mucho el verda-
  ıa,                         o        e                   ıa
dero alcance de esta estrategia. Los siguientes ejemplos ilustran lo dicho.

Problema 3.1.1 (Olimpiada Bolivariana 2000).
Sean a1 , A1 , a2 , A2 , a3 , A3 n´meros reales positivos tales que ai + Ai = k,
                                  u
donde k es una constante dada.

a) Demostrar que
                           a1 A2 + a2 A3 + a3 A1 < k 2 .
Tallerde resolucionproblemas
Tallerde resolucionproblemas
Tallerde resolucionproblemas
Tallerde resolucionproblemas
Tallerde resolucionproblemas
Tallerde resolucionproblemas
Tallerde resolucionproblemas
Tallerde resolucionproblemas
Tallerde resolucionproblemas
Tallerde resolucionproblemas
Tallerde resolucionproblemas
Tallerde resolucionproblemas
Tallerde resolucionproblemas
Tallerde resolucionproblemas
Tallerde resolucionproblemas
Tallerde resolucionproblemas
Tallerde resolucionproblemas
Tallerde resolucionproblemas
Tallerde resolucionproblemas
Tallerde resolucionproblemas
Tallerde resolucionproblemas
Tallerde resolucionproblemas
Tallerde resolucionproblemas
Tallerde resolucionproblemas
Tallerde resolucionproblemas
Tallerde resolucionproblemas
Tallerde resolucionproblemas
Tallerde resolucionproblemas
Tallerde resolucionproblemas
Tallerde resolucionproblemas
Tallerde resolucionproblemas
Tallerde resolucionproblemas
Tallerde resolucionproblemas
Tallerde resolucionproblemas
Tallerde resolucionproblemas
Tallerde resolucionproblemas
Tallerde resolucionproblemas
Tallerde resolucionproblemas
Tallerde resolucionproblemas

Contenu connexe

Tendances

Sistema de posicionamiento de objetos mediante visión estéreo embarcable en v...
Sistema de posicionamiento de objetos mediante visión estéreo embarcable en v...Sistema de posicionamiento de objetos mediante visión estéreo embarcable en v...
Sistema de posicionamiento de objetos mediante visión estéreo embarcable en v...Jorge Tarlea
 
Aprendiendo a aprender. resolver problemas con iguales
Aprendiendo a aprender. resolver problemas con igualesAprendiendo a aprender. resolver problemas con iguales
Aprendiendo a aprender. resolver problemas con igualesRossyPalmaM Palma M
 
Lisa Anderson Umana Spanish Translation Study 0n Perspectivism And Leadership...
Lisa Anderson Umana Spanish Translation Study 0n Perspectivism And Leadership...Lisa Anderson Umana Spanish Translation Study 0n Perspectivism And Leadership...
Lisa Anderson Umana Spanish Translation Study 0n Perspectivism And Leadership...Christian Camping International
 
Cap1y2.Textoguiapdf
Cap1y2.TextoguiapdfCap1y2.Textoguiapdf
Cap1y2.TextoguiapdfMonica Diaz
 
Clase 12 -analisis_de_datos
Clase 12 -analisis_de_datosClase 12 -analisis_de_datos
Clase 12 -analisis_de_datosAbraham Tafur
 
Metodologia de la investigacion sampieri hernandez-r-cap3-planteamiento-del-p...
Metodologia de la investigacion sampieri hernandez-r-cap3-planteamiento-del-p...Metodologia de la investigacion sampieri hernandez-r-cap3-planteamiento-del-p...
Metodologia de la investigacion sampieri hernandez-r-cap3-planteamiento-del-p...zenayda suarez
 
Metodologia de la investigacion
Metodologia de la investigacionMetodologia de la investigacion
Metodologia de la investigaciondanielramy
 
Les tic al_servei_de_la_promocio_economica_local_infonomia
Les tic al_servei_de_la_promocio_economica_local_infonomiaLes tic al_servei_de_la_promocio_economica_local_infonomia
Les tic al_servei_de_la_promocio_economica_local_infonomiaAlex Moga i Vidal
 
Proyecto hongos comestibles1
Proyecto hongos comestibles1Proyecto hongos comestibles1
Proyecto hongos comestibles1Sofiagalindez
 
Guia de mediacion laboral_IAFJSR
Guia de mediacion laboral_IAFJSRGuia de mediacion laboral_IAFJSR
Guia de mediacion laboral_IAFJSRMauri Rojas
 

Tendances (17)

Sistema de posicionamiento de objetos mediante visión estéreo embarcable en v...
Sistema de posicionamiento de objetos mediante visión estéreo embarcable en v...Sistema de posicionamiento de objetos mediante visión estéreo embarcable en v...
Sistema de posicionamiento de objetos mediante visión estéreo embarcable en v...
 
Aprendiendo a aprender. resolver problemas con iguales
Aprendiendo a aprender. resolver problemas con igualesAprendiendo a aprender. resolver problemas con iguales
Aprendiendo a aprender. resolver problemas con iguales
 
Lisa Anderson Umana Spanish Translation Study 0n Perspectivism And Leadership...
Lisa Anderson Umana Spanish Translation Study 0n Perspectivism And Leadership...Lisa Anderson Umana Spanish Translation Study 0n Perspectivism And Leadership...
Lisa Anderson Umana Spanish Translation Study 0n Perspectivism And Leadership...
 
Guia de matematicas
Guia de matematicasGuia de matematicas
Guia de matematicas
 
Matematicas dos primer parte
Matematicas dos primer parteMatematicas dos primer parte
Matematicas dos primer parte
 
Cap1y2.Textoguiapdf
Cap1y2.TextoguiapdfCap1y2.Textoguiapdf
Cap1y2.Textoguiapdf
 
Clase 12 -analisis_de_datos
Clase 12 -analisis_de_datosClase 12 -analisis_de_datos
Clase 12 -analisis_de_datos
 
Metodologia de la investigacion sampieri hernandez-r-cap3-planteamiento-del-p...
Metodologia de la investigacion sampieri hernandez-r-cap3-planteamiento-del-p...Metodologia de la investigacion sampieri hernandez-r-cap3-planteamiento-del-p...
Metodologia de la investigacion sampieri hernandez-r-cap3-planteamiento-del-p...
 
Metodologia de la investigacion
Metodologia de la investigacionMetodologia de la investigacion
Metodologia de la investigacion
 
Calculo diferencial
Calculo diferencialCalculo diferencial
Calculo diferencial
 
Cartilla Matemática
Cartilla MatemáticaCartilla Matemática
Cartilla Matemática
 
Les tic al_servei_de_la_promocio_economica_local_infonomia
Les tic al_servei_de_la_promocio_economica_local_infonomiaLes tic al_servei_de_la_promocio_economica_local_infonomia
Les tic al_servei_de_la_promocio_economica_local_infonomia
 
Matematicas financieras 1
Matematicas financieras 1Matematicas financieras 1
Matematicas financieras 1
 
Proyecto hongos comestibles1
Proyecto hongos comestibles1Proyecto hongos comestibles1
Proyecto hongos comestibles1
 
Impacto 4
Impacto 4Impacto 4
Impacto 4
 
Fundamentos de-calculo
Fundamentos de-calculoFundamentos de-calculo
Fundamentos de-calculo
 
Guia de mediacion laboral_IAFJSR
Guia de mediacion laboral_IAFJSRGuia de mediacion laboral_IAFJSR
Guia de mediacion laboral_IAFJSR
 

En vedette

Int pricing factors and strategies
Int pricing factors and strategiesInt pricing factors and strategies
Int pricing factors and strategiesshyamkanji
 
Speech Recognition by Iqbal
Speech Recognition by IqbalSpeech Recognition by Iqbal
Speech Recognition by IqbalIqbal
 
trabajo de investigacion: el alcoholismo en los jovenes
trabajo de investigacion: el alcoholismo en los jovenestrabajo de investigacion: el alcoholismo en los jovenes
trabajo de investigacion: el alcoholismo en los jovenesAgostina Agüero
 
Relajacion jacobson
Relajacion jacobsonRelajacion jacobson
Relajacion jacobsonkeiserps
 
32 ways to make your blog suck less
32 ways to make your blog suck less32 ways to make your blog suck less
32 ways to make your blog suck lessScott Hanselman
 

En vedette (7)

Int pricing factors and strategies
Int pricing factors and strategiesInt pricing factors and strategies
Int pricing factors and strategies
 
Speech Recognition by Iqbal
Speech Recognition by IqbalSpeech Recognition by Iqbal
Speech Recognition by Iqbal
 
Pga 2012 2013 ceip la concepción
Pga 2012 2013 ceip la concepciónPga 2012 2013 ceip la concepción
Pga 2012 2013 ceip la concepción
 
Mozart Biografia
Mozart BiografiaMozart Biografia
Mozart Biografia
 
trabajo de investigacion: el alcoholismo en los jovenes
trabajo de investigacion: el alcoholismo en los jovenestrabajo de investigacion: el alcoholismo en los jovenes
trabajo de investigacion: el alcoholismo en los jovenes
 
Relajacion jacobson
Relajacion jacobsonRelajacion jacobson
Relajacion jacobson
 
32 ways to make your blog suck less
32 ways to make your blog suck less32 ways to make your blog suck less
32 ways to make your blog suck less
 

Similaire à Tallerde resolucionproblemas

Notasrm094
Notasrm094Notasrm094
Notasrm094ivan
 
Notasrm094
Notasrm094Notasrm094
Notasrm094ivan
 
Calculo diferencial integral_func_una_var (1)
Calculo diferencial integral_func_una_var (1)Calculo diferencial integral_func_una_var (1)
Calculo diferencial integral_func_una_var (1)Roberto Soto
 
Probabilidad y estadistica elementales
Probabilidad y estadistica elementalesProbabilidad y estadistica elementales
Probabilidad y estadistica elementalesNoelia Larsen
 
Probabilidad y estadistica elementales
Probabilidad y estadistica elementalesProbabilidad y estadistica elementales
Probabilidad y estadistica elementalesSamantha Piña
 
Del Docente Presencial Al Docente Virtual
Del Docente Presencial Al Docente VirtualDel Docente Presencial Al Docente Virtual
Del Docente Presencial Al Docente Virtualmarlonint45
 
Aprendiendo a aprender. resolver problemas con iguales
Aprendiendo a aprender. resolver problemas con igualesAprendiendo a aprender. resolver problemas con iguales
Aprendiendo a aprender. resolver problemas con igualesRossyPalmaM Palma M
 
Mi proyecto de pre-tesis
Mi proyecto de pre-tesisMi proyecto de pre-tesis
Mi proyecto de pre-tesisErwin Salas
 
Dificultades-en-el-aprendizaje.pdf
Dificultades-en-el-aprendizaje.pdfDificultades-en-el-aprendizaje.pdf
Dificultades-en-el-aprendizaje.pdfaltagraciaperez15
 
Manual propedeutico-final-2014
Manual propedeutico-final-2014Manual propedeutico-final-2014
Manual propedeutico-final-2014Daniel Rivera
 
Apuntes Coello Coello 2009
Apuntes Coello Coello 2009Apuntes Coello Coello 2009
Apuntes Coello Coello 2009Ernesto Cortés
 

Similaire à Tallerde resolucionproblemas (20)

Tabla de contenido y resumen
Tabla de contenido y resumenTabla de contenido y resumen
Tabla de contenido y resumen
 
Notasrm094
Notasrm094Notasrm094
Notasrm094
 
Notasrm094
Notasrm094Notasrm094
Notasrm094
 
Calculo diferencial integral_func_una_var (1)
Calculo diferencial integral_func_una_var (1)Calculo diferencial integral_func_una_var (1)
Calculo diferencial integral_func_una_var (1)
 
Probabilidad y estadistica elementales
Probabilidad y estadistica elementalesProbabilidad y estadistica elementales
Probabilidad y estadistica elementales
 
Probabilidad y estadistica elementales
Probabilidad y estadistica elementalesProbabilidad y estadistica elementales
Probabilidad y estadistica elementales
 
Del Docente Presencial Al Docente Virtual
Del Docente Presencial Al Docente VirtualDel Docente Presencial Al Docente Virtual
Del Docente Presencial Al Docente Virtual
 
Libro logica
Libro logicaLibro logica
Libro logica
 
Aprendiendo a aprender. resolver problemas con iguales
Aprendiendo a aprender. resolver problemas con igualesAprendiendo a aprender. resolver problemas con iguales
Aprendiendo a aprender. resolver problemas con iguales
 
Didáctica de la Matemática
Didáctica de la MatemáticaDidáctica de la Matemática
Didáctica de la Matemática
 
Metodos numericos con matlab
Metodos numericos con matlabMetodos numericos con matlab
Metodos numericos con matlab
 
Mi proyecto de pre-tesis
Mi proyecto de pre-tesisMi proyecto de pre-tesis
Mi proyecto de pre-tesis
 
Texto de matematicas 2009-2011
Texto de matematicas 2009-2011Texto de matematicas 2009-2011
Texto de matematicas 2009-2011
 
Dificultades-en-el-aprendizaje.pdf
Dificultades-en-el-aprendizaje.pdfDificultades-en-el-aprendizaje.pdf
Dificultades-en-el-aprendizaje.pdf
 
Algebra[ufro]
Algebra[ufro]Algebra[ufro]
Algebra[ufro]
 
Manual propedeutico-final-2014
Manual propedeutico-final-2014Manual propedeutico-final-2014
Manual propedeutico-final-2014
 
Diseño de Experimentos
Diseño de Experimentos Diseño de Experimentos
Diseño de Experimentos
 
Algoritmos
AlgoritmosAlgoritmos
Algoritmos
 
TGT_1182 (2).pdf
TGT_1182 (2).pdfTGT_1182 (2).pdf
TGT_1182 (2).pdf
 
Apuntes Coello Coello 2009
Apuntes Coello Coello 2009Apuntes Coello Coello 2009
Apuntes Coello Coello 2009
 

Tallerde resolucionproblemas

  • 1. Talleres de Formaci´n Matem´tica o a Maracaibo, 26 al 31 de julio de 2004 Resoluci´n de o Problemas Matem´ticos a Jos´ Heber Nieto Said e
  • 2. Prefacio Estas notas constituyen el material de apoyo de un taller para estudiantes Licenciatura en Matem´ticas dirigido a desarrollar la habilidad para resolver a problemas. Aunque por lo general los problemas juegan un rol importante en cual- quier curso de matem´tica y la habilidad para resolverlos es un aspecto a importante de la evaluaci´n, los profesores suelen centrar sus esfuerzos en o los aspectos t´cnicos espec´ e ıficas de su asignatura y no en los aspectos gene- rales de la resoluci´n de problemas. El objetivo de esta obra en cambio es o ayudar al lector a desarrollar su habilidad general para resolver problemas. Es bueno dejar en claro que el desarrollo de esta habilidad es b´sica- a mente el resultado del trabajo personal, de la pr´ctica adquirida resolviendo a problemas y de la reflexi´n sobre esa pr´ctica. No es posible convertirse en o a un solucionista experto mediante la mera lectura pasiva de un libro, del mismo modo que no es posible convertirse en un buen nadador o pianista simplemente leyendo un manual. Sin embargo el conocimiento de las t´cni- e cas apropiadas y de los errores t´ ıpicos que es preciso evitar puede ser tan util para el solucionista como lo es para el nadador o el pianista. ´ Con el fin de que la obra sea de utilidad para el mayor n´mero posible u de estudiantes se ha procurado que los problemas analizados no requieran de conocimientos especializados. Sin embargo las mismas t´cnicas y estra- e tegias que ejemplificamos con problemas elementales se aplican a los m´s a avanzados.
  • 3. ´ Indice general Introducci´n o 1 1. Principios Generales 3 1.1. Resoluci´n de Problemas y Creatividad . . . o . . . . . . . . . 3 1.1.1. Invertir el problema . . . . . . . . . . . . . . . . . . . 4 1.1.2. Pensamiento lateral . . . . . . . . . . . . . . . . . . . 4 1.1.3. Principio de discontinuidad . . . . . . . . . . . . . . . 5 1.1.4. Imitaci´n . . . . . . . . . . . . . . . . o . . . . . . . . . 5 1.1.5. Tormenta de cerebros (Brainstorming) . . . . . . . . . 5 1.1.6. Mapas mentales . . . . . . . . . . . . . . . . . . . . . 5 1.1.7. Programaci´n neuroling¨´ o uıstica (PNL) . . . . . . . . . 6 1.1.8. Factores afectivos . . . . . . . . . . . . . . . . . . . . . 6 1.1.9. Bloqueos mentales . . . . . . . . . . . . . . . . . . . . 6 1.2. La Creaci´n Matem´tica . . . . . . . . . . . . o a . . . . . . . . . 7 1.3. La metodolog´ de P´lya . . . . . . . . . . . . ıa o . . . . . . . . . 8 1.4. El trabajo de Alan Schoenfeld . . . . . . . . . . . . . . . . . . 11 2. Ejemplos sencillos 14 ´ 2.1. Aritm´tica y Algebra . . . . . . . . . . . . . . . . . . . . . . . 14 e 2.2. Combinatoria . . . . . . . . . . . . . . . . . . . . . . . . . . . 19 2.3. Geometr´ . . . . . . . . . . . . . . . . . . . . . . . . . . . . . 21 ıa 3. Algunas Estrategias B´sicas a 26 3.1. Figuras y diagramas . . . . . . . . . . . . . . . . . . . . . . . 26 3.2. Examen de casos especiales . . . . . . . . . . . . . . . . . . . 28 3.3. Transformaciones e Invariantes . . . . . . . . . . . . . . . . . 30 3.4. El Principio Extremal . . . . . . . . . . . . . . . . . . . . . . 34 iii
  • 4. 4. Un problema y varias soluciones 37 4.1. Inducci´n . . . . . . . . . . . . . . o . . . . . . . . . . . . . . . 38 4.2. Teor´ de grafos . . . . . . . . . . . ıa . . . . . . . . . . . . . . . 38 4.3. Pruebas por Integraci´n . . . . . . o . . . . . . . . . . . . . . . 40 4.4. El m´todo de perturbaciones . . . e . . . . . . . . . . . . . . . 41 4.5. Funciones escalonadas . . . . . . . . . . . . . . . . . . . . . . 42 4.6. Triangulaciones y Lema de Sperner . . . . . . . . . . . . . . . 43 5. Problemas para pensar 45 6. Soluciones y sugerencias 51 Bibliograf´ ıa 60 iv
  • 5. Introducci´n o La palabra problema proviene del griego πρoβαλλειν, “lanzar adelante”. Un problema es un obst´culo arrojado ante la inteligencia para ser superado, a una dificultad que exige ser resuelta, una cuesti´n que reclama ser aclarada. o Todos vivimos resolviendo problemas: desde el m´s b´sico de asegurar la a a cotidiana subsistencia, com´n a todos los seres vivos, hasta los m´s com- u a plejos desaf´ planteados por la ciencia y la tecnolog´ La importancia de ıos ıa. la actividad de resoluci´n de problemas es evidente; en definitiva, todo el o progreso cient´ıfico y tecnol´gico, el bienestar y hasta la supervivencia de la o especie humana dependen de esta habilidad. No es de extra˜ar por lo tanto n que la misma se haya convertido en un nuevo objeto de estudio, atrayendo por igual la atenci´n de psic´logos, ingenieros, matem´ticos, especialistas o o a en inteligencia artificial y cient´ ıficos de todas las disciplinas. En el campo educativo se ha reconocido ampliamente su importancia. y en muchas Uni- versidades el desarrollo de la creatividad y de la habilidad para resolver problemas es una parte integral del curriculum. Pero lamentablemente todav´ es muy com´n que se expongan ante el ıa u alumno los productos y resultados de la resoluci´n de problemas, pero no el o proceso mismo. Si examinamos un libro de texto con problemas resueltos de matem´tica, encontraremos por lo general soluciones tersas y acabadas. a Rara vez el autor incluye comentarios sobre los intentos fallidos de soluci´n, o los casos particulares examinados antes de llegar a la soluci´n general o los o refinamientos realizados a una primera soluci´n no totalmente satisfactoria. o Estos y otros elementos del proceso son cuidadosamente eliminados y lo que se nos presenta es el producto final, conciso y elegante. Hay muchas posibles razones para que esto sea as´ un estilo de exposici´n matem´tica consa- ı: o a grado por la tradici´n, criterios est´ticos de concisi´n y elegancia, razones o e o econ´micas de las editoriales, etc. Pero la consecuencia es que el estudiante o obtiene una visi´n falseada de lo que es resolver problemas y de la actividad o 1
  • 6. 2 matem´tica en general. a Si tiene la suerte de tener un profesor que entienda y valore el proceso de resolver problemas entonces las actividades de aula suplir´n las deficiencias a del texto. Pero si no es as´ y el profesor sigue al libro al pie de la letra, al ı enfrentarse al primer fracaso el estudiante terminar´ frustrado, perder´ la a a confianza en s´ mismo y creer´ que la resoluci´n de problemas es una acti- ı a o vidad incomprensible, accesible solamente a unos pocos superdotados. Nuestro principal objetivo en esta obra es ayudar al lector a desarrollar su habilidad para resolver problemas. Es bueno dejar claro desde el principio que el desarrollo de esta habilidad es el resultado del trabajo personal, de la pr´ctica adquirida resolviendo problemas y de la reflexi´n sobre esa pr´ctica. a o a No es posible convertirse en un solucionista experto mediante la mera lectura pasiva de un libro, del mismo modo que no es posible convertirse en un buen nadador o pianista simplemente leyendo. Sin embargo el conocimiento de las t´cnicas apropiadas y de los errores t´ e ıpicos que es preciso evitar puede ser tan util para el solucionista como lo es para el nadador o el pianista. ´
  • 7. Cap´ ıtulo 1 Principios Generales “La principal raz´n de existir del matem´tico es re- o a solver problemas, y por lo tanto en lo que realmente consisten las matem´ticas es en problemas y solu- a ciones.” Paul R. Halmos [14] En este cap´ ıtulo nos ocuparemos de los m´todos y principios generales e que resultan utiles para la resoluci´n de problemas. Pero recordemos que la ´ o unica manera de aprender a resolver problemas es . . . resolviendo problemas! ´ Por lo tanto la lectura de este cap´ ıtulo solamente ser´ util si se combina con a´ la pr´ctica constante. Para quienes tengan poca experiencia es recomendable a pasar r´pidamente por las p´ginas siguientes, para volver a ellas m´s tarde, a a a como referencia, mientras est´n trabajando en la resoluci´n de problemas e o concretos. Quienes se interesen por el estudio en profundidad de la habilidad para resolver problemas pueden consultar [27]. 1.1. Resoluci´n de Problemas y Creatividad o Evidentemente la resoluci´n de problemas est´ estrechamente relaciona- o a da con la creatividad, que algunos definen precisamente como la habilidad para generar nuevas ideas y solucionar todo tipo de problemas y desaf´ ıos. La especie humana es creativa por naturaleza. Todo ser humano nace con un gran potencial para la creaci´n, pero mientras algunos lo aprovechan o al m´ximo, otros casi no lo utilizan. Sin embargo la creatividad, al igual que a
  • 8. 4 Principios Generales cualquier otra habilidad humana, puede desarrollarse a trav´s de la pr´ctica e a y el entrenamiento adecuado. Lamentablemente tambi´n puede atrofiarse, si e no se ejercita adecuadamente. El pensamiento creativo se ha dividido en divergente y convergente. El primero consiste en la habilidad para pensar de manera original y elabo- rar nuevas ideas, mientras que el segundo se relaciona con la capacidad cr´ıtica y l´gica para evaluar alternativas y seleccionar la m´s apropiada. o a Evidentemente ambos tipos de pensamiento juegan un rol fundamental en la resoluci´n de problemas. o Tres aspectos de la creatividad han recibido mucha atenci´n: el proceso o creativo, las caracter´ ısticas de la personalidad creativa, y las circunstancias que posibilitan o favorecen el acto creativo. Como consecuencia de estos es- tudios se han desarrollado t´cnicas y m´todos generales dirigidos a desarro- e e llar el potencial creativo. En esta obra nos concentraremos en las t´cnicas e y estrategias espec´ıficas que han demostrado ser m’s utiles para la resolu- ´ ci´n de problemas matem´ticos. Sin embargo haremos a continuaci´n una o a o breve rese˜a de algunos de los m´todos m´s generales, remitiendo al lector n e a interesado a la bibliograf´ correspondiente. ıa 1.1.1. Invertir el problema Cada concepto tiene uno contrario y la oposici´n entre ellos genera una o tensi´n favorable al hecho creativo. Esta idea, que tiene profundas ra´ o ıces tanto en la filosof´ oriental como en la occidental, se refleja en la sabidur´ ıa ıa popular en aforismos tales como: “Para saber mandar hay que aprender a obedecer” o “Para ser un buen orador hay que saber escuchar”. Como ejemplo de esta t´cnica supongamos que deseamos dise˜ar un zapato que e n sea muy c´modo. El problema inverso ser´ dise˜ar un zapato inc´modo. o ıa n o Pero el an´lisis de este problema nos llevar´ seguramente a descubrir los a a factores que causan incomodidad, y al evitarlos habremos dado un buen paso hacia la soluci´n del problema original. Vea [38]. o 1.1.2. Pensamiento lateral Consiste en explorar alternativas inusuales o incluso aparentemente ab- surdas para resolver un problema. En otras palabras: evitar los caminos trillados, intentar lo que nadie ha intentado, ensayar percepciones y puntos de vista diferentes. Vea [5].
  • 9. 1.1 Resoluci´n de Problemas y Creatividad o 5 1.1.3. Principio de discontinuidad La rutina suprime los est´ ımulos necesarios para el acto creativo, por lo tanto si experimenta un bloqueo temporal de su capacidad creadora in- terrumpa su programa cotidiano de actividades y haga algo diferente a lo acostumbrado. Vaya a dar un paseo por sitios que no conoce, ensaye una nueva receta de cocina, escuche m´sica diferente a la que escucha habi- u tualmente, lea un libro que no ten´ pensado leer, asista a alg´n tipo de ıa u espect´culo diferente a sus favoritos. a 1.1.4. Imitaci´n o La mayor parte de los grandes artistas comienzan imitando a sus maes- tros. M´s a´n se ha llegado a afirmar, en parte en broma y en parte en serio, a u que “la originalidad no es otra cosa que un plagio no detectado”. En cual- quier caso es claro que la imitaci´n puede ser un primer paso v´lido hacia o a la originalidad. En particular observe y no vacile en imitar las t´cnicas de e resoluci´n de problemas empleadas con ´xito por sus compa˜eros, maestros o e n o colegas. 1.1.5. Tormenta de cerebros (Brainstorming) Es una t´cnica desarrollada en el mundo de la publicidad, en el cual el e ´xito depende de la generaci´n de nuevas y brillantes ideas. Para ello se e o re´ne un grupo de personas y se les invita a expresar todas las ideas que u se les ocurran en relaci´n a un problema o tema planteado, sin importar lo o estrafalarias o rid´ ıculas que parezcan. La evaluaci´n y la cr´ o ıtica se posponen, esperando crear un clima estimulante que favorezca el surgimiento de algunas ideas realmente utiles. La utilidad de esta t´cnica es dudosa fuera de ciertos ´ e campos o situaciones muy espec´ ıficas. 1.1.6. Mapas mentales Es una t´cnica desarrollada por Tony Buzan (vea [6] y [7]) que trata de e representar en forma gr´fica el car´cter asociativo de la mente humana. Se a a comienza con la idea principal ubicada en el centro de la hoja y alrededor de ella se van colocando las ideas asociadas y sus respectivos v´ ınculos. Uti- lizando diversos colores y s´ ımbolos esta t´cnica puede llegar a ser muy util e ´ para organizar las ideas que van surgiendo en torno a un problema.
  • 10. 6 Principios Generales 1.1.7. Programaci´n neuroling¨´ o uıstica (PNL) Tambi´n conocida como “la ciencia de la experiencia subjetriva”, es un e conjunto de t´cnicas muy desarrolladas a trav´s de las cuales se trata de e e caracterizar el contexto (f´ ısico, fisiol´gico, psicol´gico, ambiental, etc.) en o o el cual somos m´s creativos, para luego reproducirlo a voluntad. Los prac- a ticantes de la PNL han incluso “modelado” el comportamiento de algunos personajes famosos, tales como Walt Disney, para tratar de aprovechar sus modos y procedimientos m´s creativos. Vea [10] y [11]. a 1.1.8. Factores afectivos La resoluci´n de problemas no es un asunto puramente intelectual. Las o emociones, y en particular el deseo de resolver un problema, tienen tambi´n e una gran importancia. La incapacidad que manifiestan algunos alumnos para resolver incluso el ejercicio m´s sencillo no es producto por lo general de una a deficiencia intelectual, sino de una absoluta falta de inter´s y motivaci´n. A e o veces no existe ni siquiera el deseo de comprender el problema, y por lo tanto el mismo no es comprendido. El profesor que desee realmente ayudar a un alumno con estas caracter´ ısticas deber´ ante todo despertarle su curiosidad a dormida, motivarlo y transmitirle deseos de logro y superaci´n. o Algunas creencias negativas para el proceso creativo est´n asociadas a a una baja autoestima y pueden tener ra´ emocionales profundas. Por ejem- ıces plo hay quienes enfrentados a un problema creen a priori que no podr´n a resolverlo, y que si lo intentan s´lo conseguir´n terminar con un dolor de o a cabeza. El maestro o profesor debe en estos casos apelar a todas sus dotes y conocimientos como educador, aunque en casos extremos ser´ necesaria a tambi´n la ayuda de un orientador o la de un psic´logo. e o En el polo opuesto, alguien que tenga confianza en su propia capacidad y crea que un problema es un desaf´ que vale la pena enfrentar y que re- ıo solverlo le proporcionar´ una satisfacci´n intelectual al mismo tiempo que a o ser´ una experiencia valiosa para su formaci´n, estar´ en excelentes condi- a o a ciones psicol´gicas para abordar el proceso resolutivo. Para profundizar en o estos aspectos vea [4], [24], [25], [26]. 1.1.9. Bloqueos mentales James Adams, profesor de dise˜o en la Universidad de Stanford, centra su n enfoque de la creatividad en la superaci´n de los bloqueos mentales, barreras o
  • 11. 1.2 La Creaci´n Matem´tica o a 7 que nos impiden percibir un problema en la forma correcta y encontrarle soluci´n. En [1] analiza diferentes tipos de bloqueos y propone ejercicios o para identificarlos y superarlos. Su clasificaci´n es la siguiente: o Bloqueos perceptivos: estereotipos, dificultad para aislar el proble- ma, delimitar demasiado el espacio de soluciones, imposibilidad de ver el problema desde varios puntos de vista, saturaci´n, no poder utilizar o toda la informaci´n sensorial. o Bloqueos emocionales: miedo a cometer errores, a arriesgar, a fra- casar; deseo de seguridad y orden; preferir juzgar ideas a concebirlas; inhabilidad para relajarse; falta de est´ ımulo; entusiasmo excesivo; falta de control imaginativo. Bloqueos culturales: tab´es; el peso de la tradici´n; roles predeter- u o minados asignados a la mujer y al hombre. Bloqueos ambientales: distracciones; falta de apoyo para llevar ade- lante una idea; falta de cooperaci´n entre colegas. o Bloqueos intelectuales: inhabilidad para seleccionar un lenguaje apropiado para el problema (verbal, matem´tico, visual); uso inade- a cuado de las estrategias; falta de informaci´n o informaci´n incorrecta. o o Bloqueos expresivos: t´cnicas inadecuadas para registrar y expresar e ideas (a los dem´s y a uno mismo) a 1.2. La Creaci´n Matem´tica o a Una de las reflexiones m´s profundas que se han hecho sobre la creativi- a dad en matem´tica es la realizada a principios de siglo por Henri Poincar´, a e uno de los m´s grandes matem´ticos de su tiempo. En una conferencia pro- a a nunciada ante la Sociedad Psicol´gica de Par´ [30] hizo interesant´ o ıs ısimas revelaciones sobre sus propias experiencias como creador: “¿Qu´ es, de hecho, la creaci´n matem´tica? No consiste en e o a hacer combinaciones nuevas con entes matem´ticos ya conoci- a dos. Cualquiera podr´ hacerlo, pcro las combinaciones que se ıa podr´ hacer as´ ser´ un n´mero limitado y en su mayor´ ıan ı ıan u ıa totalmente desprovistas de inter´s. Crear consiste precisamente e
  • 12. 8 Principios Generales no en construir las combinaciones in´tiles, sino en construir las u que son utiles y que est´n en ´ ´ a ınfima minor´ Crear es discernir, ıa. es escoger. . . ” “A menudo, cuando se trabaja en un problema dif´ ıcil, no se consigue nada la primera vez que se comienza la tarea. Luego se toma un descanso m´s o menos largo y uno se sienta de nuevo a ante la mesa. Durante la primera media hora se contin´a sin u encontrar nada. Despu´s, de repente. la idea decisiva se presenta e ante la mente. . . ” “Hay que hacer otra observaci´n a prop´sito de las condicio- o o nes de este trabajo inconsciente. Se trata de que tal trabajo no es posible, y en todo caso no es fecundo, si no est´ por una parte a precedido y por otra seguido de un per´ ıodo de trabajo conscien- te. Estas inspiraciones s´bitas no se presentan . . . m´s que tr´s u a a algunos d´ de esfuerzos voluntarios, aparentemente est´riles, en ıas e los que uno ha cre´ no hacer nada interesante, y piensa haber ıdo tomado un camino falso totalmente. Estos esfuerzos no fueron, por tanto, tan est´riles como se pensaba. Pusieron en movimien- e to la m´quina inconsciente y sin ellos ´sta no habr´ funcionado a e ıa ni hubiera producido nada. . . ” Poincar´ esboza luego una teor´ del trabajo del yo subliminal, en la e ıa cual atribuye un rol fundamental a la sensibilidad y el sentido est´tico del e matem´tico en el proceso de selecci´n, durante el trabajo inconsciente, de a o las combinaciones m´s significativas. a Una conclusi´n pr´ctica: cuando un problema se resiste a nuestros mejo- o a res esfuerzos, nos queda todav´ la posibilidad de dejarlo durante un tiempo, ıa descansar, dar un paseo, y volver a ´l m´s tarde. Sin embargo solamente e a aquellos problemas que nos han apasionado, manteni´ndonos en una con- e siderable tensi´n mental, son los que vuelven m´s tarde, transformados, a o a la mente consciente. La inspiraci´n o iluminaci´n s´bita, que los antiguos o o u consideraban un don divino, hay que merecerla. 1.3. La metodolog´ de P´lya ıa o En 1945 el insigne matem´tico y educador George P´lya (1887–1985) a o public´ un libro que r´pidamente se convirtir´ en un cl´sico: How to solve o a ıa a it [31]. En el mismo propone una metodolog´ en cuatro etapas para resolver ıa
  • 13. 1.3 La metodolog´ de P´lya ıa o 9 problemas. A cada etapa le asocia una serie de preguntas y sugerencias que aplicadas adecuadamente ayudar´n a resolver el problema. Las cuatro etapas a y las preguntas a ellas asociadas se detallan a continuaci´n: o Etapa I: Comprensi´n del problema. o ¿Cu´l es la inc´gnita? ¿Cu´les son los datos? ¿Cual es la condici´n? a o a o ¿Es la condici´n suficiente para determinar la inc´gnita? ¿Es insufi- o o ciente? ¿Redundante? ¿Contradictoria? Etapa II: Concepci´n de un plan. o ¿Se ha encontrado con un problema semejante? ¿Ha visto el mismo problema planteado en forma ligeramente diferente? ¿Conoce un problema relacionado con ´ste? ¿Conoce alg´n teorema e u que le pueda ser util? Mire atentamente la inc´gnita y trate de recordar ´ o un problema que le sea familiar y que tenga la misma inc´gnita o una o inc´gnita similar. o He aqu´ un problema relacionado con el suyo y que se ha resuelto ya. ı ¿Podr´ utilizarlo? ¿Podr´ emplear su resultado? ¿Podr´ utilizar su ıa ıa ıa m´todo? ¿Podr´ utilizarlo introduciendo alg´n elemento auxiliar? e ıa u ¿Podr´ enunciar el problema en otra forma? ¿Podr´ plantearlo en ıa ıa forma diferente nuevamente? Refi´rase a las definiciones. e Si no puede resolver el problema propuesto, trate de resolver primero alg´n problema similar. ¿Podr´ imaginarse un problema an´logo un u ıa a tanto m´s accesible? ¿Un problema m´s general? ¿Un problema m´s a a a particular? ¿Un problema an´logo? ¿Puede resolver una parte del pro- a blema? Considere s´lo una parte de la condici´n; descarte la otra parte; o o ¿en qu´ medida la inc´gnita queda ahora determinada? ¿en qu´ forma e o e puede variar? ¿Puede usted deducir alg´n elemento util de los datos? u ´ ¿Puede pensar en algunos otros datos apropiados para determinar la inc´gnita? ¿Puede cambiar la inc´gnita? ¿Puede cambiar la inc´gnita o o o o los datos, o ambos si es necesario, de tal forma que la nueva inc´gnita o y los nuevos datos est´n m´s cercanos entre s´ e a ı? ¿Ha empleado todos los datos? ¿Ha empleado toda la condici´n? ¿Ha o considerado usted todas las nociones esenciales concernientes al pro- blema?
  • 14. 10 Principios Generales Etapa III: Ejecuci´n del plan. o Al ejecutar el plan, compruebe cada uno de los pasos. ¿Puede ver claramente que el paso es correcto? ¿Puede demostrarlo? Etapa IV. Visi´n retrospectiva. o ¿Puede usted verificar el resultado? ¿Puede verificar el razonamiento? ¿Puede obtener el resultado en forma diferente? ¿Puede verlo de golpe? ¿Puede emplear el resultado o el m´todo en alg´n otro problema? e u La primera etapa es obviamente insoslayable: es imposible resolver un problema del cual no se comprende el enunciado. Sin embargo en nuestra pr´ctica como docentes hemos visto a muchos estudiantes lanzarse a efectuar a operaciones y aplicar f´rmulas sin reflexionar siquiera un instante sobre lo o que se les pide. Por ejemplo si en el problema aparece una funci´n comienzan o de inmediato a calcularle la derivada, independientemente de lo que diga el enunciado. Si el problema se plantea en un examen y luego, comentando los resultados, el profesor dice que el c´lculo de la derivada no se ped´ y m´s a ıa a a´n que el mismo era irrelevante para la soluci´n del problema, algunos le u o responder´n: ¿o sea que no nos va a dar ning´n punto por haber calculado a u la derivada? Este tipo de respuesta revela una incomprensi´n absoluta de o lo que es un problema y plantea una situaci´n muy dif´ al profesor, quien o ıcil tendr´ que luchar contra vicios de pensamiento arraigados, adquiridos tal a vez a lo largo de muchos a˜os. n La segunda etapa es la m´s sutil y delicada, ya que no solamente est´ re- a a lacionada con los conocimientos y la esfera de lo racional, sino tambi´n con e la imaginaci´n y la creatividad. Observemos que las preguntas que P´lya o o asocia a esta etapa est´n dirigidas a llevar el problema hacia un terreno co- a nocido. Con todo lo utiles que estas indicaciones son, sobre todo para el tipo ´ de problemas que suele presentarse en los cursos ordinarios, dejan planteada una interrogante: ¿qu´ hacer cuando no es posible relacionar el problema e con algo conocido? En este caso no hay recetas infalibles, hay que trabajar duro y confiar en nuestra propia creatividad e inspiraci´n. o La tercera etapa es de car´cter m´s t´cnico. Si el plan est´ bien concebi- a a e a do, su realizaci´n es factible y poseemos los conocimientos y el entrenamiento o necesarios, deber´ ser posible llevarlo a cabo sin contratiempos. Sin embar- ıa go por lo general en esta etapa se encontrar´n dificultades que nos obligar´n a a
  • 15. 1.4 El trabajo de Alan Schoenfeld 11 a regresar a la etapa anterior para realizar ajustes al plan o incluso para modificarlo por completo. Este proceso puede reperirse varias veces. La cuarta etapa es muchas veces omitida, incluso por solucionistas exper- tos. P´lya insiste mucho en su importancia, no solamente porque comprobar o los pasos realizados y verificar su correcci´n nos puede ahorrar muchas sor- o presas desagradables, sino porque la visi´n retrospectiva nos puede conducir o a nuevos resultados que generalicen, ampl´ o fortalezcan el que acabamos ıen de hallar. 1.4. El trabajo de Alan Schoenfeld Si bien la mayor´ de los matem´ticos reconocen en las estrategias heur´ ıa a ıs- ticas de P´lya los m´todos que ellos mismos utilizan habitualmente, no es o e tan f´cil para el que no tiene experiencia aplicarlas exitosamente. En otras a palabras, dichas estrategias son m´s descriptivas que prescriptivas. Alan a Schoenfeld (ver [34], [35], [36]) es uno de los que m´s han estudiado esta a problem´tica. En su an´lisis identifica los siguientes cuatro factores relevan- a a tes para la resoluci´n de problemas: o Recursos cognitivos. Son nuestros conocimientos matem´ticos ge- a nerales, tanto de conceptos y resultados como de procedimientos (al- goritmos). Heur´ ıstica. Es el conjunto de estrategias y t´cnicas para resolver e problemas que conocemos y estamos en capacidad de aplicar. Control o metacognici´n. Es la capacidad de utilizar lo que sabe- o mos para lograr un objetivo. Creencias. Se refiere a aquellas creencias y opiniones relacionadas con la resoluci´n de problemas y que pueden afectarla favorable o o desfavorablemente. La importancia del primer factor es obvia. Sin embargo se ha demostra- do (ver [9]) que no es suficiente poseer un amplio bagaje de conocimientos matem´ticos para ser un solucionista experto. Tambi´n es necesario dominar a e algunas t´cnicas y estrategias que nos ayuden a atacar el problema. En do- e minios restringidos y bien delimitados, en los cuales los problemas a resolver son m´s o menos rutinarios, se han desarrollado estrategias que pueden ser a
  • 16. 12 Principios Generales aplicadas con ´xito incluso por un computador, con resultados tan buenos o e mejores que los obtenidos por los expertos humanos (estos son los famosos sistemas expertos, producto de las investigaciones en inteligencia artificial y ciencia cognitiva). Sin embargo para resolver problemas no rutinarios en dominios ricos en contenido, como la matem´tica, se requiere algo m´s que a a conocimientos y estrategias. Ese factor adicional es lo que llamamos con- trol; act´a como una voz interior que nos dice qu´ ideas y estrategias (entre u e muchas alternativas posibles) nos conviene aplicar para el problema que te- nemos entre manos, o bien si debemos abandonar un camino que no parece arrojar resultados o por el contrario redoblar esfuerzos y perseverar en ´l. e Los solucionistas inexpertos tienen evidentes deficiencias en este aspecto: se apresuran a transitar el primer camino que se les ocurre y luego se mueven en c´ırculos, cayendo una y otra vez en el mismo error. El ultimo factor puede influir tambi´n de manera importante en el pro- ´ e ceso de resoluci´n de problemas. Algunas creencias comunes, sobre todo o entre estudiantes de ense˜anza media, son las siguientes: “todo problema n se resuelve mediante alguna f´rmula”, “lo importante es el resultado y no o el procedimiento”, “la respuesta del libro no puede estar equivocada”. Este tipo de creencias es un obst´culo para el desempe˜o de cualquier persona a n como solucionista. Schoenfeld elabor´ tambi´n una lista de las estrategias m´s utilizadas: o e a 1. An´lisis. a a) Dibuje un diagrama siempre que sea posible. b) Examine casos especiales. 1) Seleccione algunos valores especiales para ejemplificar el pro- blema e irse familiarizando con ´l. e 2) Examine casos l´ ımite para explorar el rango de posibilidades. 3) Si hay un par´metro entero, dele sucesivamente los valores a 1, 2, . . . , m y vea si emerge alg´n patr´n inductivo. u o c) Trate de simplificar el problema. 1) Explotando la existencia de simetr´ ıa. 2) Usando argumentos del tipo “sin p´rdida de generalidad”. e 2. Exploraci´n. o a) Considere problemas esencialmente equivalentes.
  • 17. 1.4 El trabajo de Alan Schoenfeld 13 1) Reemplazando condiciones por otras equivalentes. 2) Recombinando los elementos del problema de maneras dife- rentes. 3) Introduciendo elementos auxiliares. 4) Reformulando el problema: Mediante un cambio de perspectiva o notaci´n. o Mediante argumentos por contradicci´n o contraposici´n. o o Asumiendo que tenemos una soluci´n y determinando sus o propiedades. b) Considere un problema ligeramente modificado. 1) Escoja submetas (tratando de satisfacer parcialmente las con- diciones). 2) Relaje una condici´n y luego trate de reimponerla. o 3) Descomponga el dominio del problema y trabaje caso por caso. c) Considere problemas sustancialmente modificados. 1) Construya un problema an´logo con menos variables. a 2) Deje todas las variables fijas excepto una, para determinar su impacto. 3) Trate de aprovechar cualquier problema relacionado que ten- ga forma, datos o conclusiones similares. 3. Verificaci´n de la soluci´n. o o a) ¿Pasa su soluci´n estas pruebas espec´ o ıficas? 1) ¿Usa todos los datos pertinentes? 2) ¿Est´ de acuerdo con estimaciones o predicciones razonables? a 3) ¿Soporta pruebas de simetr´ an´lisis dimensional y escala? ıa, a b) ¿Pasa estas pruebas generales? 1) ¿Puede ser obtenida de manera diferente? 2) ¿Puede ser sustanciada por casos especiales? 3) ¿Puede ser reducida a resultados conocidos? 4) ¿Puede utilizarse para generar alg´n resultado conocido? u
  • 18. Cap´ ıtulo 2 Ejemplos sencillos “Resolver un problema es hacer un descubrimiento. Un gran problema significa un gran descubrimien- to, pero hay una part´ıcula de descubrimiento en la soluci´n de cualquier problema. El suyo puede ser o modesto, pero si pone a prueba la curiosidad que induce a poner en juego las facultades inventivas, y si lo resuelve por medios propios, puede experimen- tar la tensi´n y el encanto del descubrimiento y el o goce del triunfo.” George P´lya [31] o En este cap´ıtulo pondremos en pr´ctica los principios examinados en el a cap´ıtulo anterior. Para ello hemos seleccionado varios problemas sencillos y de f´cil soluci´n, de modo que nos podamos concentrar en el proceso de a o resoluci´n m´s que en el contenido de los mismos. o a 2.1. ´ Aritm´tica y Algebra e Algunos de los problemas m´s antiguos que se conocen son de tipo a aritm´tico. Es t´ e ıpico que se pida hallar una cantidad determinada por cier- tas condiciones, o bien efectuar un reparto cumpliendo ciertos requisitos. Los siguientes problemas pertenecen a esta categor´ ıa. Problema 2.1. Diofanto fue un notable matem´tico griego que desarroll´ su a o actividad en Alejandr´ en el siglo III A.C. y del cual se conservan muy pocos ıa
  • 19. e ´ 2.1 Aritm´tica y Algebra 15 datos biogr´ficos. Sin embargo se dice que su epitafio conten´ la siguiente a ıa inscripci´n: o Caminante: aqu´ yacen los restos de Diofanto. Y los n´me- ı u ros pueden mostrar cu´n larga fue su vida, cuya sexta parte a constituy´ su hermosa infancia. Hab´ transcurrido adem´s una o ıa a duod´cima parte cuando sus mejillas se cubrieron de vello. Lue- e go de una s´ptima parte se cas´, y transcurrido un quinquenio e o le hizo dichoso el nacimiento de su primog´nito, cuya existencia e dur´ tan s´lo la mitad de la de su padre. Luego de cuatro a˜os o o n buscando consuelo en la ciencia de los n´meros, descendi´ Dio- u o fanto a la sepultura. ¿Qu´ edad alcanz´ Diofanto? ¿A qu´ edad se cas´? ¿Cu´ntos a˜os vivi´ su e o e o a n o hijo? Soluci´n. Veamos si comprendemos bien el problema. ¿Cu´l es la inc´gnita? o a o El n´mero de a˜os que vivi´ Diofanto (las preguntas restantes se responden u n o f´cilmente conociendo la respuesta a la primera). ¿Cu´les son los datos? Una a a serie de informaciones sobre las etapas sucesivas de su vida, desde su infan- cia hasta su muerte. Ahora debemos concebir un plan. ¿Se ha encontrado con un problema semejante? Es de esperar que s´ ya que la mayor´ de los ı, ıa problemas resolubles por m´todos algebraicos elementales son semejantes. e El plan general consiste en escribir ecuaciones que reflejen las condiciones planteadas, resolver el sistema resultante y finalmente interpretar las solu- ciones obtenidas en el contexto original del problema. Llamemos x al n´mero u de a˜os vividos por Diofanto. Esta cantidad debe ser igual a la suma de las n duraciones de las etapas de su vida, a saber: su infancia (x/6), la duod´cima e parte transcurrida hasta que le sali´ barba (x/12), los a˜os transcurridos o n hasta que contrajo matrimonio (x/7), los a˜os transcurridos hasta que na- n ci´ su primog´nito (5), los a˜os que ´ste vivi´ (x/2) y los 4 a˜os que Diofanto o e n e o n le sobrevivi´. Por lo tanto escribimos: o x x x x x= + + +5+ +4 (2.1) 6 12 7 2 Agrupando t´rminos semejantes resulta: e 1 1 1 1 (1 − − − − )x = 5 + 4 6 12 7 2 y simplificando queda 3 x = 9. 28
  • 20. 16 Ejemplos sencillos Por lo tanto x = 28 × 9/3 = 84. Verifiquemos el resultado: 84 84 84 84 + + +5+ + 4 = 14 + 7 + 12 + 5 + 42 + 4 = 84 6 12 7 2 Diofanto se cas´ cuando contaba 84/6 + 84/12 + 84/7 = 33 a˜os, y su hijo o n vivi´ 84/2 = 42 a˜os. o n Los documentos matem´ticos m´s antiguos que se conservan son dos a a rollos de papiro egipcios que datan aproximadamente de la XII dinast´ ıa (2078 a 1788 A.C.). Uno de ellos, conocido como el papiro Rhind, consta de unos 85 problemas y ejemplos pr´cticos. El siguiente es uno de ellos: a Problema 2.2. Dividir cien panes entre cinco hombres, de modo que las porciones que reciban est´n en progresi´n aritm´tica y que la s´ptima parte e o e e de la suma de las tres mayores sea igual a la suma de las dos porciones menores. Soluci´n. Asegur´monos de comprender bien el problema. ¿Qu´ se nos pide? o e e Dividir cien panes entre cinco hombres, de modo que se cumplan ciertas con- diciones. ¿Cu´les son los datos? El n´mero total de panes (100), la cantidad a u de porciones (5) y las condiciones que debe cumplir el reparto. ¿Cu´les son a las inc´gnitas? Obviamente, la cantidad de panes que le corresponder´ a ca- o a da uno. ¿Comprendemos la condici´n? En primer lugar las porciones deben o estar en progresi´n aritm´tica; esto significa que si escribimos las porciones o e en orden creciente de magnitud, la diferencia de cada una de ellas con la siguiente es constante. En otras palabras, si llamamos x a la menor de las porciones y r a la diferencia com´n o raz´n de la progresi´n, entonces las u o o cinco porciones deber´n ser x, x + r, x + 2r, x + 3r y x + 4r. Utilizando esta a notaci´n podemos describir la ultima condici´n del problema mediante una o ´ o ecuaci´n: o (x + 2r) + (x + 3r) + (x + 4r) = x + (x + r) (2.2) 7 ¿Es la condici´n suficiente para determinar la inc´gnita? ¿Es insuficiente? o o Estas preguntas vienen muy bien en este momento, ya que nos hacen ob- servar que tenemos dos inc´gnitas x y r pero una sola ecuaci´n. En general o o (pero por supuesto hay excepciones) esto significa que el problema es inde- terminado, es decir que en vez de una unica soluci´n admite varias, tal vez ´ o hasta un n´mero infinito de ellas. Pero otra posibilidad a tener en cuenta u es que no tengamos suficientes ecuaciones sencillamente por haber pasado
  • 21. e ´ 2.1 Aritm´tica y Algebra 17 por alto alg´n dato o condici´n del problema. Recordemos las preguntas u o de P´lya: ¿Ha empleado todos los datos?, ¿Ha empleado toda la condici´n? o o Bueno, leyendo una vez m´s el enunciado del problema vemos que no hemos a utilizado el hecho de que los panes a dividir son cien. Este dato nos permite escribir otra ecuaci´n: o x + (x + r) + (x + 2r) + (x + 3r) + (x + 4r) = 100 (2.3) Bien, ya tenemos dos ecuaciones y dos inc´gnitas. El plan a seguir es simple: o resolver el sistema. Para ello simplificamos primero las ecuaciones 2.2 y 2.3 hasta obtener 11x − 2r = 0 (2.4) x + 2r = 20 (2.5) de donde resulta x = 5/3 y r = 55/6. Las cinco porciones ser´n entonces: a 5/3 = 1 2 , 5/3 + 55/6 = 65/6 = 10 5 , 65/6 + 55/6 = 20, 20 + 55/6 = 175/6 = 3 6 29 1 y finalmente 175/6 + 55/6 = 115/3 = 38 1 . 6 3 Visi´n retrospectiva: ¿Puede usted verificar el resultado? Esto es f´cil: 5/3 + o a 65/6 + 20 + 175/6 + 115/3 = 100 y 65/6 − 5/3 = 20 − 65/6 = 175/6 − 20 = 115/3 − 175/6 = 55/6. ¿Puede obtener el resultado en forma diferente? Bueno, si se tiene cierta experiencia resolviendo problemas con progresiones aritm´ticas se observa que muchas veces resulta m´s c´modo representar la e a o progresi´n de manera sim´trica, alrededor de un t´rmino central. En nuestro o e e caso, si llamamos z al t´rmino central y r a la raz´n, los cinco t´rminos ser´n e o e a z − 2r, z − r, z, z + r y z + 2r. Ahora la condici´n de que las partes suman o cien se escribe as´ı: (z − 2r + +(z − r) + z + (z + r) + (z + 2r) = 100 que se reduce a 5z = 100 y por tanto z = 20. La otra condici´n es o 20 + (20 + r) + (20 + 2r) = (20 − 2r) + (20 − r) 7 que luego de simplificar nos da 60 + 3r = 7(40 − 3r), de donde podemos despejar r = (280 − 60)/24 = 55/6. Obtenemos por supuesto la misma soluci´n que antes, pero el procedimiento luce m´s limpio y elegante: en lugar o a de resolver un sistema de dos ecuaciones con dos inc´gnitas s´lo tenemos que o o resolver un par de ecuaciones de primer grado. Esto se debe a que la simetr´ıa hace que se cancelen los t´rminos con r en la primera ecuaci´n. e o
  • 22. 18 Ejemplos sencillos Problema 2.3. Tres recipientes contienen agua. Si se vierte 1/3 del conte- nido del primer recipiente en el segundo, y a continuaci´n 1/4 del contenido o del segundo en el tercero, y por ultimo 1/10 del contenido del tercero en el ´ primero, entonces cada recipiente queda con 9 litros de agua. ¿Qu´ cantidad e de agua hab´ originalmente en cada recipiente? ıa Soluci´n. Este problema puede tratarse en principio con el mismo m´todo o e que los anteiores: si llamamos x, y, z a los contenidos iniciales de los recipien- tes es posible escribir unas ecuaciones que reflejen las condiciones del pro- blema. Por ejemplo, despu´s de la primera operaci´n el contenido del primer e o recipiente ser´ (2/3)x y el del segundo y + x/3. Luego de la segunda opera- a ci´n el contenido del segundo recipiente ser´ (3/4)(y + x/3) = x/4 + (3/4)y o a y el del tercero z + (1/4)(y + x/3) = x/12 + y/4 + z. Luego de la tercera operaci´n el contenido del tercer recipiente ser´ (9/10)(x/12 + y/4 + z) y el o a del primero (2/3)x + (1/10)(x/12 + y/4 + z). Igualando ahora el contenido final de cada recipiente con 9 obtenemos un sistema de tres ecuaciones con tres inc´gnitas, cuya soluci´n es la respuesta buscada. Los detalles se los o o dejamos al lector como ejercicio. Visi´n retrospectiva: No cabe duda de que el m´todo anterior, aunque in- o e falible, es bastante aburrido y proclive a errores num´ricos. ¿No habr´ otra e a forma de proceder m´s apropiada para este tipo de problema? S´ la hay, a ı y consiste en sustituir el an´lisis hacia adelante que realizamos, partiendo a de la configuraci´n inicial y estudiando la evoluci´n del contenido de los o o recipientes con cada operaci´n, por un an´lisis retrospectivo. Este tipo de o a an´lisis consiste en partir de la configuraci´n final y estudiar c´mo se lleg´ a a o o o ella. En nuestro caso los tres recipientes finalizan con 9 litros, y la ultima ´ operaci´n consisti´ en trasvasar 1/10 del contenido del tercer recipiente al o o primero. Pero si el tercer recipiente, luego de perder la d´cima parte de su e contenido, qued´ con 9 litros, es obvio que deb´ contener diez litros. Y el o ıa primero, como qued´ con 9 luego de ganar un litro, antes conten´ 8 litros. o ıa En otras palabras, despu´s de la segunda operaci´n y antes de la tercera el e o contenido de los recipientes era 8, 9 y 10 litros, en ese orden. Del mismo mo- do se ve que antes de la segunda operaci´n el segundo recipiente conten´ 12 o ıa litros, para poder quedar en 9 al perder la cuarta parte de su contenido. Y el tercero, por consiguiente, ten´ 7 litros. Los contenidos antes de la segunda ıa operaci´n eran entonces 8, 12 y 7. Razonando de igual forma llegamos a que o inicialmente los recipientes conten´ 12, 8 y 10 litros de agua. Este an´lisis ıan a retrospectivo se resume en la siguiente tabla:
  • 23. 2.2 Combinatoria 19 1◦ 2◦ 3◦ 9 9 9 8 9 10 8 12 7 12 8 10 2.2. Combinatoria Hay una clase importante de problemas en los cuales tenemos que contar o enumerar configuraciones resultantes de combinar, de alguna manera, un n´mero finito de elementos. La rama de la matem´tica que se ocupa de esto u a se conoce como combinatoria. Los siguientes son algunos ejemplos sencillos. Problema 2.4. Un cubo s´lido de madera de lado 20 cm se pinta de rojo. o Luego con una sierra se hacen cortes paralelos a las caras, de cent´ ımetro en ımetro, hasta obtener 203 = 8000 cubitos de lado 1 cm. ¿Cu´ntos de cent´ a esos cubitos tendr´n al menos una cara pintada de rojo? a Soluci´n. El problema es de f´cil comprensi´n. El primer plan que se nos o a o ocurre es sencillamente contar los cubitos pintados. Por ejemplo: en cada cara del cubo hay 202 = 400 cubitos pintados, por lo tanto en total ser´n a . . . ¿400×6? No, porque estar´ıamos contando m´s de una vez los cubitos que a est´n en los v´rtices y aristas del cubo. Pero al menos esto nos da una pista a e para mejorar el plan (y una cota superior: el n´mero de cubitos pintados u debe ser menor que 2400). Contemos entonces por separado los diferentes tipos de cubitos pintados: Los correspondientes a los v´rtices del cubo, que tienen tres caras e pintadas y son ocho en total. Los correspondientes a las aristas del cubo, exclu´ ıdos los v´rtices (tie- e nen exactamente dos caras pintadas). Cada arista tiene contacto con 20 cubitos, pero dos de ellos son v´rtices (que ya contamos aparte) por e lo cual nos quedan 18. Como el cubo tiene 12 aristas, el n´mero total u es 18 × 12 = 216. Los cubitos con exactamente una cara pintada. En cada cara del cubo, las caras pintadas de estos cubitos forman un cuadrado de 18 × 18, por lo tanto en total ser´n 18 × 18 × 6 = 1944. a
  • 24. 20 Ejemplos sencillos Por consiguiente la respuesta es 8 + 216 + 1944 = 2168. Visi´n retrospectiva: ¿Podemos obtener el resultado en forma diferente? Una o primera alternativa es partir de nuestro primer resultado err´neo, 2400, y o efectuar las correcciones necesarias. Como los cubos de los v´rtices se con- e taron tres veces cada uno, restemos 8 × 2 = 16. Y como los de las aristas se contaron dos veces, restemos 216. El resultado ser´ 2400 − 16 − 216 = 2168. a Otra idea (posiblemente la m´s elegante) se obtiene invirtiendo el proble- a ma. Contemos los cubitos que no tienen ninguna cara pintada. Es claro que estos cubitos forman un cubo interior al primero, de lado 18. Por lo tanto son 183 = 5832. Los que tiene al menos una cara pintada se pueden obtener ahora restando esta ultima cantidad del total de cubitos, a saber ´ 203 − 183 = 8000 − 5832 = 2168. Problema 2.5. En cada una de las 64 casillas de un tablero de ajedrez hay un grano de az´car. Una hormiga comienza en un v´rtice del tablero, come u e el az´car, y se traslada a una casilla adyacente, desplaz´ndose en direcci´n u a o horizontal o vertical (pero nunca en diagonal). Contin´a de este modo hasta u acabar con todo el az´car, y sin pasar dos veces por una misma casilla. ¿Es u posible que su trayecto finalice en el v´rtice diagonalmente opuesto al inicial? e Soluci´n. Este problema es de naturaleza diferente a los anteriores. No se o nos pide calcular nada, por lo cual muchos pensar´n que no es un verdadero a problema de matem´tica. Sin embargo, si hacemos abstracci´n de la hormiga a o y el az´car (que obviamente se han incluido para hacer m´s atractivo el u a enunciado) vemos que el problema trata de la existencia de trayectorias con ciertas caracter´ısticas geom´tricas. e Por alguna raz´n, la mayor´ de las personas a quienes les he planteado o ıa este problema contestan de inmediato que s´ Cuando les pido que dibujen ı. en la pizarra la trayectoria, demuestran que no han comprendido cabalmente el enunciado: trazan l´ ıneas diagonales, pasan m´s de una vez por la misma a casilla o simplemente finalizan en un v´rtice que no es el opuesto al inicial, e y a´n as´ creen haber solucionado el problema. Cuando por fin comprenden u ı las condiciones, luego de dos o tres intentos fallidos cambian s´bitamen- u te de posici´n y contestan que es imposible. Ahora bien, es claro que una o respuesta afirmativa queda suficientemente justificada con s´lo exhibir una o trayectoria que cumpla las condiciones pedidas. ¿Pero c´mo podemos jus- o tificar una respuesta negativa? Es muy importante comprender la enorme diferencia que existe entre las afirmaciones “no puedo hallar ninguna solu- ci´n” y “no existe ninguna soluci´n”. Para poder afirmar esto ultimo hay o o ´
  • 25. 2.3 Geometr´ ıa 21 b´sicamente dos maneras de proceder. Una de ellas consiste en dibujar todas a las trayectorias posibles que parten de un v´rtice y recorren todo el tablero, e desplaz´ndose en direcci´n horizontal o vertical y sin pasar dos veces por a o ninguna casilla. Una vez hecho esto podemos examinar las trayectorias y verificar que ninguna finaliza en el v´rtice opuesto al inicial. Un inconve- e niente de este procedimiento es que resulta muy lento y engorroso para un ser humano, aunque ser´ factible realizarlo con ayuda del computador. Otro ıa inconveniente es que si se nos ocurre generalizar el problema para tableros m´s grandes r´pidamente el problema se vuelve inmanejable, incluso para a a el computador. M´s a´n, si queremos una respuesta general, para tableros a u de n × n, este procedimiento resulta completamente in´til. u La segunda manera de proceder es demostrar que no existe trayectoria alguna que cumpla las condiciones exigidas. Para esto resulta util el hecho ´ de que las casillas de un tablero de ajedrez est´n pintadas de dos colores, a digamos blanco y negro, en forma alternada. La observaci´n clave es que o cada movimiento unitario en direcci´n horizontal o vertical nos lleva de una o casilla a otra de diferente color. Ahora bien, como el tablero tiene 8 × 8 = 64 casillas, comenzando en cualquiera de ellas se requieren 63 movimientos para recorrerlas todas. Pero es claro que despu´s de 1, 3, 5 o cualquier n´mero e u impar de movimientos estaremos en una casilla de color diferente a la inicial. Esto demuestra que la respuesta al problema que nos ocupa es negativa, ya que un v´rtice y el opuesto son del mismo color. e Visi´n retrospectiva: Una generalizaci´n obvia de este problema consiste en o o considerar tableros de n × n, para cualquier entero positivo n. Es claro que si n es par entonces la respuesta es negativa, por el mismo argumento usado para el caso 8 × 8. En cambio si n es impar el argumento no se aplica. De hecho es f´cil ver que la respuesta es afirmativa. Otras generalizaciones que a se resuelven con el mismo m´todo: especificar dos casillas cualesquiera como e inicio y fin de la trayectoria; cambiar el tipo de movimiento b´sico, usando a por ejemplo saltos de caballo; plantear el problema en tres dimensiones, por ejemplo en un cubo. El lector interesado en estos temas puede consultar [29]. 2.3. Geometr´ ıa La otra clase importante de problemas que encontramos en la matem´ti- a ca elemental son los de geometr´ El lector interesado en este tema puede ıa.
  • 26. 22 Ejemplos sencillos consultar [12]. Hay una gran variedad de problemas geom´tricos: problemas de construc- e ci´n, de c´lculo, de demostraci´n, etc. El siguiente es un ejemplo sencillo. o a o Problema 2.6. Los lados del tri´ngulo ABC miden AB = 26cm, BC = a 17cm y CA = 19cm. Las bisectrices de los ´ngulos de v´rtices B y C se a e cortan en el punto I. Por I se traza una paralela a BC que corta a los lados AB y BC en los puntos M y N respectivamente. Calcule el per´ ımetro del tri´ngulo AM N . a Soluci´n. La primera de las estrategias que Schoenfeld coloca en su lista o es hacer un diagrama, toda vez que sea posible. Si bien esta recomendaci´n o se aplica a todo tipo de problemas, es casi insoslayable si el problema es de car´cter geom´trico. Muchas veces el enunciado de estos problemas va a e acompa˜ado de un dibujo, pero otras veces (como en este caso) no es as´ n ı, y hacerlo es la primera tarea que debemos realizar. Tal vez usted haya o´ ıdo frases tales como “un dibujo no constituye demostraci´n”, “razonar en base o a un dibujo puede conducir a errores”, etc. Todo eso es cierto, sin embargo un dibujo nos ayuda en primer lugar a comprender el problema. Adem´s a estimular´ nuestra imaginaci´n y es posibleque nos sugiera alg´n plan para a o u hallar la soluci´n. Si tiene a mano instrumentos geom´tricos uselos; sin em- o e ´ bargo incluso un bosquejo aproximado suele ser de mucha ayuda (¡H´galo a antes de seguir leyendo!). Hay muchas maneras de resolver este problema. El que tenga afici´n a los o c´lculos complicados podr´ por ejemplo comenzar por hallar el ´rea del a ıa a tri´ngulo ABC (usando la f´rmula de Heron). Dividiendo el ´rea entre el a o a semiper´ımetro se obtiene el radio de la circunferencia inscripta, es decir la distancia de I a los lados del tri´ngulo ABC. Con estos datos es posible a
  • 27. 2.3 Geometr´ ıa 23 calcular, por proporcionalidad, las longitudes de AM , M N y AN . Sin em- bargo esto es bastante engorroso. ¿No habr´ una manera m´s sencilla? Si a a miramos el dibujo detenidamente, buscando alguna relaci´n interesante, ob- o servaremos (sobre todo si el dibujo est´ bien hecho) que los tri´ngulos BM I a a y CN I parecen is´sceles. Si esto fuese cierto la soluci´n ser´ inmediata, ya o o ıa que de las igualdades M I = M B y IN = N C se obtiene: AM + M N + AN = AM + M I + IN + AN = AM + M B + AN + N C = AB + AC = 26 + 19 = 45. Ahora bien, ¿podremos probar que los tri´ngulos BM I y CN I son is´sce- a o les? Para probar por ejemplo que BM I es is´sceles es suficiente probar que o los ´ngulos ∠M BI y ∠M IB son iguales. Pero sabemos que M N es paralela a a BC, por lo tanto ∠M IB = ∠IBC ya que son ´ngulos alternos internos. a Pero BI es la bisectriz de ∠ABC, por lo tanto ∠M BI = ∠IBC y hemos completado la demostraci´n (por supuesto que para el tri´ngulo CN I se o a razona de modo an´logo). a Visi´n retrospectiva: Si revisamos los datos del problema vemos que hay o uno de ellos que no fue utilizado: la longitud del lado BC. En realidad para cualquier tri´ngulo con AB = 26 cm y CA = 19 la soluci´n ser´ la misma, a o ıa 26 + 19 = 45. ¿Y si variamos AB y CA? Bueno, es f´cil ver que la respuesta a ser´ siempre AB + CA. En otras palabras, los valores 26 y 19 no juegan a ning´n papel especial, y mucho menos BC = 17. Estos datos en vez de ayu- u dar a resolver el problema m´s bien estorban, dirigiendo nuestra atenci´n a o hacia detalles sin importancia. Son elementos distractores , que aumentan la dificultad del problema suministrando m´s informaci´n que la estrictamente a o necesaria para resolverlo. Para aclarar mejor este punto supongamos que el enunciado del problema hubiese sido: En un tri´ngulo ABC las bisectrices de los ´ngulos de v´rtices B a a e y C se cortan en el punto I. Por I se traza una paralela a BC que corta a los lados AB y BC en los puntos M y N respectivamente. Calcule el per´ımetro del tri´ngulo AM N en funci´n de los lados a o AB y AC. Este problema, a pesar de ser m´s general, es probablemente m´s f´cil de a a a resolver ya que nuestra atenci´n se enfocar´ directamente hacia los lados o a AB y AC. Este es el sentido de la recomendaci´n de P´lya: “eonsidere un o o problema m´s general”, la cual parece parad´jica ya que un problema m´s a o a
  • 28. 24 Ejemplos sencillos general deber´ ser por l´gica m´s dif´ Sin embargo una abstracci´n ade- ıa o a ıcil. o cuada, al eliminar la hojarasca innecesaria, puede permitirnos ver el camino con m´s claridad. Ahora bien, ¿es posible detectar y evitar el efecto de es- a tos elementos distractores? Es bastante dif´ıcil, ya que a priori no podemos saber cu´les datos son esenciales y cu´les superfluos para resolver un pro- a a blema. Sin embargo es razonable desconfiar de los datos que parecen muy particulares para la naturaleza del problema. Pero hay que tener cuidado, ya que hay propiedades que s´ dependen de valores muy particulares de los ı datos (esto es muy com´n en problemas de aritm´tica, por ejemplo). u e Muchos problemas no se pueden clasificar de manera clara dentro de una rama de la matem´tica, sino que se encuentran en la frontera entre dos o a m´s de ellas. El siguiente, por ejemplo, pertenece tanto a la geometr´ como a ıa a la combinatoria. Problema 2.7. ¿En cu´ntas regiones queda dividido el plano por 6 rectas a en posici´n gen´rica (es decir tales que no haya dos de ellas paralelas ni tres o e concurrentes en un punto)? Soluci´n. Evidentemente una recta divide el plano en dos regiones, y dos o rectas no paralelas lo dividen en cuatro. Pero ya para tres rectas el problema comienza a complicarse. Si trazamos unos cuantos diagramas veremos que la tercera recta atraviesa siempre a tres de las cuatro regiones determinadas por las dos primeras, pero no a la cuarta, y por lo tanto la respuesta para tres rectas parece ser siete. ¿Pero podemos estar seguros de esto? ¿Y qu´ pa- e sar´ cuando tracemos la cuarta, la quinta y la sexta recta? Lamentablemente a los dibujos se complican demasiado, algunas rectas se cortan fuera de la ho- ja y no es f´cil contar las regiones sin equivocarnos. Adem´s pareciera que a a la respuesta depende de como dibujemos las rectas. Volvamos entonces al principio. ¿Podr´ imaginarse un problema an´logo un tanto m´s accesible? ıa a a Bueno, en vez de disminu´ el n´mero de rectas podemos disminu´ la di- ır u ır mensi´n, es decir considerar en cu´ntas regiones queda dividida una recta o a por cierto n´mero de puntos. Este problema s´ es f´cil, n puntos dividen a u ı a la recta en n + 1 regiones (a saber n − 1 segmentos y 2 semirrectas). ¿Y no podemos aprovechar este resultado para el problema en el plano? Veamos, si ya hemos trazado n − 1 rectas entonces al trazar la n-sima ´sta cortar´ a e a las anteriores en n − 1 puntos diferentes (por la hi´tesis de genericidad). Por o lo tanto la n-sima recta quedar´ dividida en n partes por esos puntos de a intersecci´n. Pero es claro que cada una de esas partes estar´ contenida por o a
  • 29. 2.3 Geometr´ ıa 25 completo en una regi´n de las determinadas por las primeras n − 1 rectas, o regi´n que quedar´ dividida en dos por la n-sima recta. Por lo tanto hemos o a descubierto que al trazar la n-sima recta el n´mero de regiones aumenta en u n unidades. Apliquemos ahora este resultado desde el comienzo y de mane- ra sucesiva. Inicialmente hay una sola regi´n: el plano. Al trazar la primera o recta el n´mero de regiones aumenta en una unidad, y tendremos 1 + 1 = 2 u regiones. Al trazar la segunda recta el n´mero de regiones aumenta en dos u unidades, y tendremos 2 + 2 = 4 regiones. Al trazar la tercera recta el n´me- u ro de regiones aumenta en tres unidades, y tendremos 4 + 3 = 7 regiones. Hasta aqu´ los resultados concuerdan con lo que ya sab´ ı ıamos. Ahora resulta f´cil continuar: para cuatro rectas son 7 + 4 = 11 regiones, para cinco rectas a son 11 + 5 = 16 regiones, para seis rectas son 16 + 6 = 22 regiones. Visi´n retrospectiva: Resulta natural preguntarse cu´l ser´ el n´mero de re- o a a u giones en que queda dividido el plano por un n´mero n cualquiera de rectas u en posici´n gen´rica. Recordando que la suma de los enteros desde 1 hasta o e n es n(n + 1)/2 es f´cil obtener a 1 + 1 + 2 + 3 + · · · + n = 1 + n(n + 1)/2 = (n2 + n + 2)/2 Hay otras generalizaciones y problemas similares a los cuales se puede aplicar el mismo m´todo. e
  • 30. Cap´ ıtulo 3 Algunas Estrategias B´sicas a En este cap´ıtulo se enuncian algunas estrategias b´sicas y se ilustra su a aplicaci´n a la soluci´n de varios problemas, muchos de ellos tomados de o o competencias matem´ticas internacionales. a 3.1. Figuras y diagramas El proverbio una figura vale m´s que mil palabras tiene plena validez en a la resoluci´n de problemas matem´ticos. Por eso nuestra primera estrategia o a es: Estrategia 1. Dibuje una figura o un diagrama siempre que sea posible. La importancia de este principio es obvia cuando se trata de resolver un problema de geometr´ Pero hay muchos problemas que, sin ser de geome- ıa. tr´ admiten una interpretaci´n geom´trica, lo cual ampl´ mucho el verda- ıa, o e ıa dero alcance de esta estrategia. Los siguientes ejemplos ilustran lo dicho. Problema 3.1.1 (Olimpiada Bolivariana 2000). Sean a1 , A1 , a2 , A2 , a3 , A3 n´meros reales positivos tales que ai + Ai = k, u donde k es una constante dada. a) Demostrar que a1 A2 + a2 A3 + a3 A1 < k 2 .